Matrices

Short Answer Type Questions

1. If a matrix has 28 elements, what are the possible orders it can have? What if it has 13 elements?

Show Answer

Solution

The possible orders that a matrix having 28 elements are ${28 \times 1,1 \times 28,2 \times 14,14 \times 2,4 \times 7,7 \times 4}$. The possible orders of a matrix having 13 elements are ${1 \times 13,13 \times 1}$.

2. In the matrix $A= \begin{bmatrix} a & 1 & x \\ 2 & \sqrt{3} & x^{2}-y \\ 0 & 5 & \frac{-2}{5} \end{bmatrix} $, write:

(i) The order of the matrix A (ii) The number of elements

(iii) Write elements $a _{23}, a _{31}, a _{12}$

Show Answer

Solution

(i) The order of the given matrix $A$ is $3 \times 3$

(ii) The number of elements in matrix $A=3 \times 3=9$

(iii) $a _{i j}=$ the elements of $i^{\text{th }}$ row and $j^{\text{th }}$ column.

So, $a _{23}=x^{2}-y, a _{31}=0, a _{12}=1$.

3. Construct $a _{2 \times 2}$ matrix where

(i) $a _{i j}=\frac{(i-2 j)^{2}}{2}$

(ii) $a _{i j}=|-2 i+3 j|$

Show Answer

Solution

Let $A= \begin{bmatrix} a _{11} & a _{12} \\ a _{21} & a _{22} \end{bmatrix} _{2 \times 2}$

(i) Given that $\quad a _{i j}=\frac{(i-2 j)^{2}}{2}$

$a _{11}=\frac{(1-2 \times 1)^{2}}{2}=\frac{1}{2} ; a _{12}=\frac{(1-2 \times 2)^{2}}{2}=\frac{9}{2}$

$a _{21}=\frac{(2-2 \times 1)^{2}}{2}=0 ; a _{22}=\frac{(2-2 \times 2)^{2}}{2}=2$

Hence, the matrix $A= \begin{bmatrix} \frac{1}{2} & \frac{9}{2} \\ 0 & 2 \end{bmatrix} $ (ii) Given that $a _{ij}=|-2 i+3 j|$

$ \begin{aligned} & a _{11}=|-2 \times 1+3 \times 1|=1 ; a _{12}=|-2 \times 1+3 \times 2|=4 \\ & a _{21}=|-2 \times 2+3 \times 1|=-1 ; a _{22}=|-2 \times 2+3 \times 2|=2 \end{aligned} $

Hence, the matrix $A= \begin{bmatrix} 1 & 4 \\ -1 & 2 \end{bmatrix} $

4. Construct a $3 \times 2$ matrix whose elements are given by $a _{i j}=e^{i x} \sin j x$.

Show Answer

Solution

Let

$ A= \begin{bmatrix} a _{11} & a _{12} \\ a _{21} & a _{22} \\ a _{31} & a _{32} \end{bmatrix} _{3 \times 2} $

Given that $a _{i j}=e^{i x} \sin j x$

$ \begin{matrix} a _{11}=e^{x} \sin x & a _{12}=e^{x} \sin 2 x \\ a _{21}=e^{2 x} \sin x & a _{22}=e^{2 x} \sin 2 x \\ a _{31}=e^{3 x} \sin x & a _{32}=e^{3 x} \sin 2 x \end{matrix} $

Hence, the matrix $A= \begin{bmatrix} e^{x} \sin x & e^{x} \sin 2 x \\ e^{2 x} \sin x & e^{2 x} \sin 2 x \\ e^{3 x} \sin x & e^{3 x} \sin 2 x \end{bmatrix} $

5. Find the values of $a$ and $b$ if $A=B$, where

$ A= \begin{bmatrix} a+4 & 3 b \\ 8 & -6 \end{bmatrix} , \quad B= \begin{bmatrix} 2 a+2 & b^{2}+2 \\ 8 & b^{2}-5 b \end{bmatrix} $

Show Answer

Solution

Given that $A=B$

$ \Rightarrow \quad \begin{bmatrix} a+4 & 3 b \\ 8 & -6 \end{bmatrix} = \begin{bmatrix} 2 a+2 & b^{2}+2 \\ 8 & b^{2}-5 b \end{bmatrix} $

Equating the corresponding elements, we get

$ \begin{aligned} & a+4=2 a+2, \quad 3 b=b^{2}+2 \quad \text{ and } \quad b^{2}-5 b=-6 \\ & \Rightarrow \quad 2 a-a=2, \quad b^{2}-3 b+2=0, \quad b^{2}-5 b+6=0 \\ & \therefore \quad a=2 \\ & \therefore \quad b^{2}-3 b+2=0 \\ & \therefore \quad b^{2}-5 b+6=0 \\ & \Rightarrow \quad b^{2}-2 b-b+2=0 \text{, } \\ & b^{2}-3 b-2 b+6=0 \\ & \Rightarrow b(b-2)-1(b-2)=0, \quad \Rightarrow \quad b(b-3)-2(b-3)=0 \\ & \Rightarrow \quad(b-1)(b-2)=0, \quad \Rightarrow \quad(b-2)(b-3)=0 \\ & \therefore \quad b=1,2 \Rightarrow \quad b=2,3 \end{aligned} $

but here 2 is common.

Hence, the value of $a=2$ and $b=2$.

6. If possible, find the sum of the matrices $A$ and $B$, where

$ A= \begin{bmatrix} \sqrt{3} & 1 \\ 2 & 3 \end{bmatrix} \text{ and } B= \begin{bmatrix} x & y & z \\ a & b & 6 \end{bmatrix} \text{. } $

Show Answer

Solution

The order of matrix $A=2 \times 2$ and the order of matrix $B=2 \times 3$ Addition of matrices is only possible when they have same order. So, $A+B$ is not possible.

7. If $X= \begin{bmatrix} 3 & 1 & -1 \\ 5 & -2 & -3\end{bmatrix} $ and $Y= \begin{bmatrix} 2 & 1 & -1 \\ 7 & 2 & 4 \end{bmatrix} $, find

(i) $X+Y$

(ii) $2 X-3 Y$

(iii) A matrix $Z$ such that $X+Y+Z$ is a zero matrix.

Show Answer

Solution

Given that $X= \begin{bmatrix} 3 & 1 & -1 \\ 5 & -2 & -3\end{bmatrix} $ and $Y= \begin{bmatrix} 2 & 1 & -1 \\ 7 & 2 & 4 \end{bmatrix} $

(i) $X+Y= \begin{bmatrix} 3 & 1 & -1 \\ 5 & -2 & -3\end{bmatrix} + \begin{bmatrix} 2 & 1 & -1 \\ 7 & 2 & 4 \end{bmatrix} $

$ = \begin{bmatrix} 3+2 & 1+1 & -1-1 \\ 5+7 & -2+2 & -3+4 \end{bmatrix} = \begin{bmatrix} 5 & 2 & -2 \\ 12 & 0 & 1 \end{bmatrix} $

(ii) $2 X-3 Y=2 \begin{bmatrix} 3 & 1 & -1 \\ 5 & -2 & -3\end{bmatrix} -3 \begin{bmatrix} 2 & 1 & -1 \\ 7 & 2 & 4 \end{bmatrix} $

$ \begin{aligned} & = \begin{bmatrix} 2 \times 3 & 2 \times 1 & -2 \times 1 \\ 2 \times 5 & -2 \times 2 & -2 \times 3 \end{bmatrix} - \begin{bmatrix} 3 \times 2 & 1 \times 3 & -1 \times 3 \\ 3 \times 7 & 3 \times 2 & 3 \times 4 \end{bmatrix} \\ & = \begin{bmatrix} 6 & 2 & -2 \\ 10 & -4 & -6 \end{bmatrix} - \begin{bmatrix} 6 & 3 & -3 \\ 21 & 6 & 12 \end{bmatrix} \\ & = \begin{bmatrix} 6-6 & 2-3 & -2+3 \\ 10-21 & -4-6 & -6-12 \end{bmatrix} = \begin{bmatrix} 0 & -1 & 1 \\ -11 & -10 & -18 \end{bmatrix} \end{aligned} $

(iii) $X+Y+Z=0$

$\Rightarrow \begin{bmatrix} 3 & 1 & -1 \\ 5 & -2 & -3\end{bmatrix} + \begin{bmatrix} 2 & 1 & -1 \\ 7 & 2 & 4\end{bmatrix} + \begin{bmatrix} a & b & c \\ d & e & f\end{bmatrix} = \begin{bmatrix} 0 & 0 & 0 \\ 0 & 0 & 0\end{bmatrix} $ where $\mathbf{Z}= \begin{bmatrix} a & b & c \\ d & e & f \end{bmatrix} $

$ \begin{matrix} \Rightarrow & { \begin{bmatrix} 3+2+a & 1+1+b & -1-1+c \\ 5+7+d & -2+2+e & -3+4+f \end{bmatrix} } & = \begin{bmatrix} 0 & 0 & 0 \\ 0 & 0 & 0 \end{bmatrix} \\ \Rightarrow & { \begin{bmatrix} 5+a & 2+b & -2+c \\ 12+d & e & 1+f \end{bmatrix} } & = \begin{bmatrix} 0 & 0 & 0 \\ 0 & 0 & 0 \end{bmatrix} \end{matrix} $

Equating the corresponding elements, we get

$5+a=0 \Rightarrow a=-5, \quad 2+b=0 \Rightarrow b=-2, \quad-2+c=0 \Rightarrow c=2$

$12+d=0 \Rightarrow d=-12, \quad e=0, \quad 1+f=0 \Rightarrow f=-1$

Hence, the matrix $Z= \begin{bmatrix} -5 & -2 & 2 \\ -12 & 0 & -1 \end{bmatrix} $

8. Find non-zero values of $x$ satisfying the matrix equation:

$ x \begin{bmatrix} 2 x & 2 \\ 3 & x \end{bmatrix} +2 \begin{bmatrix} 8 & 5 x \\ 4 & 4 x \end{bmatrix} =2 \begin{bmatrix} (x^{2}+8) & 24 \\ (10) & 6 x \end{bmatrix} $

Show Answer

Solution

The given equation can be written as

$ \begin{aligned} & { \begin{bmatrix} 2 x^{2} & 2 x \\ 3 x & x^{2} \end{bmatrix} + \begin{bmatrix} 16 & 10 x \\ 8 & 8 x \end{bmatrix} = \begin{bmatrix} (2 x^{2}+16) & 48 \\ 20 & 12 x \end{bmatrix} } \\ & \Rightarrow \quad \begin{bmatrix} 2 x^{2}+16 & 12 x \\ 3 x+8 & x^{2}+8 x \end{bmatrix} = \begin{bmatrix} 2 x^{2}+16 & 48 \\ 20 & 12 x \end{bmatrix} \end{aligned} $

Equating the corresponding elements we get

$ \begin{aligned} & 12 x=48, \quad 3 x+8=20, \\ & x^{2}+8 x=12 x \\ & \therefore x=\frac{48}{12}=4, \quad 3 x=20-8=12, \quad \Rightarrow \quad x^{2}=12 x-8 x=4 x \\ & \therefore \quad x=4, \quad \Rightarrow \quad x^{2}-4 x=0 \\ & x=0, x=4 \end{aligned} $

Hence, the non-zero values of $x$ is 4 .

9. If $A= \begin{bmatrix} 0 & 1 \\ 1 & 1\end{bmatrix} $ and $B= \begin{bmatrix} 0 & -1 \\ 1 & 0 \end{bmatrix} $, show that

$ (A+B)(A-B) \neq A^{2}-B^{2} $

Show Answer

Solution

Given that $A= \begin{bmatrix} 0 & 1 \\ 1 & 1\end{bmatrix} $ and $B= \begin{bmatrix} 0 & -1 \\ 1 & 0 \end{bmatrix} $

$ \begin{aligned} & A+B= \begin{bmatrix} 0 & 1 \\ 1 & 1 \end{bmatrix} + \begin{bmatrix} 0 & -1 \\ 1 & 0 \end{bmatrix} \\ & \Rightarrow \quad A+B= \begin{bmatrix} 0+0 & 1-1 \\ 1+1 & 1+0 \end{bmatrix} \Rightarrow A+B= \begin{bmatrix} 0 & 0 \\ 2 & 1 \end{bmatrix} \\ & A-B= \begin{bmatrix} 0 & 1 \\ 1 & 1 \end{bmatrix} - \begin{bmatrix} 0 & -1 \\ 1 & 0 \end{bmatrix} \\ & \Rightarrow \quad A-B= \begin{bmatrix} 0-0 & 1+1 \\ 1-1 & 1-0 \end{bmatrix} \Rightarrow A-B= \begin{bmatrix} 0 & 2 \\ 0 & 1 \end{bmatrix} \end{aligned} $

$ \begin{aligned} \therefore(A+B) \cdot(A-B)= \begin{bmatrix} 0 & 0 \\ 2 & 1 \end{bmatrix} \begin{bmatrix} 0 & 2 \\ 0 & 1 \end{bmatrix} = \begin{bmatrix} 0+0 & 0+0 \\ 0+0 & 4+1 \end{bmatrix} = \begin{bmatrix} 0 & 0 \\ 0 & 5 \end{bmatrix} \\ \begin{aligned} \text{ Now, R.H.S. } & =A^{2}-B^{2} \\ & =A \cdot A-B \cdot B \\ & = \begin{bmatrix} 0 & 1 \\ 1 & 1 \end{bmatrix} \begin{bmatrix} 0 & 1 \\ 1 & 1 \end{bmatrix} - \begin{bmatrix} 0 & -1 \\ 1 & 0 \end{bmatrix} \begin{bmatrix} 0 & -1 \\ 1 & 0 \end{bmatrix} \\ & = \begin{bmatrix} 0+1 & 0+1 \\ 0+1 & 1+1 \end{bmatrix} - \begin{bmatrix} 0-1 & 0+0 \\ 0+0 & -1+0 \end{bmatrix} \\ & = \begin{bmatrix} 1 & 1 \\ 1 & 2 \end{bmatrix} - \begin{bmatrix} -1 & 0 \\ 0 & -1 \end{bmatrix} = \begin{bmatrix} 1+1 & 1-0 \\ 1-0 & 2+1 \end{bmatrix} = \begin{bmatrix} 2 & 1 \\ 1 & 3 \end{bmatrix} \end{aligned} \end{aligned} $

Hence, $ \begin{bmatrix} 0 & 0 \\ 0 & 5\end{bmatrix} \neq \begin{bmatrix} 2 & 1 \\ 1 & 3 \end{bmatrix} $

Hence, $(A+B) \cdot(A-B) \neq A^{2}-B^{2}$

10. Find the value of $x$ if

$ \begin{bmatrix} 1 & x & 1 \end{bmatrix} \begin{bmatrix} 1 & 3 & 2 \\ 2 & 5 & 1 \\ 15 & 3 & 2 \end{bmatrix} \begin{bmatrix} 1 \\ 2 \\ x \end{bmatrix} =O $

Show Answer

Solution

Given that $ \begin{bmatrix} 1 & x & 1\end{bmatrix} \begin{bmatrix} 1 & 3 & 2 \\ 2 & 5 & 1 \\ 15 & 3 & 2\end{bmatrix} \begin{bmatrix} 1 \\ 2 \\ x \end{bmatrix} =O$

$ \begin{aligned} & \Rightarrow \begin{bmatrix} 1+2 x+15 & 3+5 x+3 & 2+x+2 \end{bmatrix} \begin{bmatrix} 1 \\ 2 \\ x \end{bmatrix} =O \\ & \Rightarrow \quad \begin{bmatrix} 2 x+16 & 5 x+6 & x+4 \end{bmatrix} \begin{bmatrix} 1 \\ 2 \\ x \end{bmatrix} =O \\ & \Rightarrow[2 x+16+10 x+12+x^{2}+4 x]=0 ; \Rightarrow x^{2}+16 x+28=0 \\ & \Rightarrow \quad x^{2}+14 x+2 x+28=0 ; \Rightarrow x(x+14)+2(x+14)=0 \\ & \Rightarrow \quad(x+2)(x+14)=0 ; x+2=0 \quad \text{ or } \quad x+14=0 \\ & \therefore \quad x=-2 \text{ or } x=-14 \end{aligned} $

Hence, the values of $x$ are -2 and -14 .

11. Show that $A= \begin{bmatrix} 5 & 3 \\ -1 & -2 \end{bmatrix} $ satisfies the equation $A^{2}-3 A-7 I=O$ and hence find $A^{-1}$.

Show Answer

Solution

Given that $A= \begin{bmatrix} 5 & 3 \\ -1 & -2 \end{bmatrix} $

$ \begin{aligned} A^{2} & =A \cdot A \\ & = \begin{bmatrix} 5 & 3 \\ -1 & -2 \end{bmatrix} \begin{bmatrix} 5 & 3 \\ -1 & -2 \end{bmatrix} = \begin{bmatrix} 25-3 & 15-6 \\ -5+2 & -3+4 \end{bmatrix} = \begin{bmatrix} 22 & 9 \\ -3 & 1 \end{bmatrix} \end{aligned} $

$A^{2}-3 A-7 I=O$

L.H.S. $ \begin{bmatrix} 22 & 9 \\ -3 & 1\end{bmatrix} -3 \begin{bmatrix} 5 & 3 \\ -1 & -2\end{bmatrix} -7 \begin{bmatrix} 1 & 0 \\ 0 & 1 \end{bmatrix} $

$\Rightarrow \begin{bmatrix} 22 & 9 \\ -3 & 1\end{bmatrix} - \begin{bmatrix} 15 & 9 \\ -3 & -6\end{bmatrix} - \begin{bmatrix} 7 & 0 \\ 0 & 7 \end{bmatrix} $

$\Rightarrow \begin{bmatrix} 22-15-7 & 9-9-0 \\ -3+3-0 & 1+6-7\end{bmatrix} \Rightarrow \begin{bmatrix} 0 & 0 \\ 0 & 0 \end{bmatrix} \quad$ R.H.S.

We are given $A^{2}-3 A-7 I=O$

$\Rightarrow \quad A^{-1}[A^{2}-3 A-7 I]=A^{-1} O$

[Pre-multiplying both sides by $A^{-1}$ ]

$\Rightarrow \quad A^{-1} A \cdot A-3 A^{-1} \cdot A-7 A^{-1} I=O$

$[A^{-1} O=O]$

$\Rightarrow \quad I \cdot A-3 I-7 A^{-1} I=O$

$\Rightarrow A-3 I-7 A^{-1}=O$

$\Rightarrow \quad-7 A^{-1}=3 I-A$

$\Rightarrow \quad A^{-1}=\frac{1}{-7}[3 I-A]$

$\Rightarrow \quad A^{-1}=\frac{1}{-7}[3(\begin{matrix} 1 & 0 \\ 0 & 1\end{matrix} )-(\begin{matrix} 5 & 3 \\ -1 & -2\end{matrix} )]$

$=\frac{1}{-7}[(\begin{matrix} 3 & 0 \\ 0 & 3\end{matrix} )-(\begin{matrix} 5 & 3 \\ -1 & -2\end{matrix} )]$

$=\frac{1}{-7} \begin{bmatrix} 3-5 & 0-3 \\ 0+1 & 3+2\end{bmatrix} =\frac{1}{-7} \begin{bmatrix} -2 & -3 \\ 1 & 5 \end{bmatrix} $

Hence, $A^{-1}=-\frac{1}{7} \begin{bmatrix} -2 & -3 \\ 1 & 5 \end{bmatrix} $

12. Find the matrix A satisfying the matrix equation:

$ \begin{bmatrix} 2 & 1 \\ 3 & 2 \end{bmatrix} A \begin{bmatrix} -3 & 2 \\ 5 & -3 \end{bmatrix} = \begin{bmatrix} 1 & 0 \\ 0 & 1 \end{bmatrix} $

Show Answer

Solution

Let $A= \begin{bmatrix} a & b \\ c & d \end{bmatrix} _{2 \times 2}$

$ \begin{aligned} { \begin{bmatrix} 2 & 1 \\ 3 & 2 \end{bmatrix} _{2 \times 2} \begin{bmatrix} a & b \\ c & d \end{bmatrix} _{2 \times 2} \begin{bmatrix} -3 & 2 \\ 5 & -3 \end{bmatrix} _{2 \times 2} } & = \begin{bmatrix} 1 & 0 \\ 0 & 1 \end{bmatrix} _{2 \times 2} \\ \Rightarrow \quad \begin{bmatrix} 2 a+c & 2 b+d \\ 3 a+2 c & 3 b+2 d \end{bmatrix} _{2 \times 2} \begin{bmatrix} -3 & 2 \\ 5 & -3 \end{bmatrix} _{2 \times 2} & = \begin{bmatrix} 1 & 0 \\ 0 & 1 \end{bmatrix} _{2 \times 2} \\ \Rightarrow \quad \begin{bmatrix} -6 a-3 c+10 b+5 d & 4 a+2 c-6 b-3 d \\ -9 a-6 c+15 b+10 d & 6 a+4 c-9 b-6 d \end{bmatrix} & = \begin{bmatrix} 1 & 0 \\ 0 & 1 \end{bmatrix} \end{aligned} $

Equating the corresponding elements, we get,

$-6 a-3 c+10 b+5 d =1\qquad$……(1)

$-9 a-6 c+15 b+10 d =0\qquad$……(2)

$4 a+2 c-6 b-3 d =0\qquad$……(3)

$6 a+4 c-9 b-6 d =1\qquad$……(4)

Multiplying eq. (1) by 2 and subtracting eq. (2), we get,

$$ \begin{matrix} -12 a-6 d+20 b+10 d=2 \\ 9 a-6 c+15 b+10 d=0 & \\ (+)\quad(+)\quad(-)\quad(-)\quad(-) \\ \hline-3a\qquad +5b\qquad =2 \end{matrix} $$

$ \begin{aligned} & -3 a+5 b=2 \end{aligned} $

Now, multiplying eq. (3) by 2 and subtracting eq. (4), we get

$$ \begin{matrix} 8 a+4 c /-12 b-6 d /=0 \\ 6 a+4 d-9 b-6 d=1 \\ (-) \quad(-) \quad(+) \quad(+)\quad (-) \\ \hline 2 a \qquad-3 b\qquad=-1 \tag{6}\\ 2 a-3 b=-1 & \end{matrix} $$

Solving eq. (5) and (6) i.e.,

$ \begin{aligned} -3 a+5 b & =2 \\ 2 a-3 b & =-1 \\ \Rightarrow \quad-6 a+10 b & =4 \\ \Rightarrow \quad 6 a-9 b & =-3 \\ \text{ Adding } \quad b & =1 \end{aligned} $

$ \begin{matrix} 2 \times(-3 a+5 b=2) & \Rightarrow & -6 a+10 b= & 4 \\ 3 \times(2 a-3 b=-1) & \Rightarrow & 6 a-9 b=-3 \end{matrix} $

Putting the value of $b$ in eq. (6), we get,

$ 2 a-3 \times 1=-1 $

$\Rightarrow 2 a-3=-1 \Rightarrow 2 a=3-1 \Rightarrow 2 a=2$

$\therefore \quad a=1$

Now, putting the values of $a$ and $b$ in equations (1) and (3)

$ \begin{aligned} & -6 \times 1-3 c+10 \times 1+5 d=1 \\ & \Rightarrow \quad-6-3 c+10+5 d=1 \end{aligned} $

$$ \begin{equation*} \Rightarrow \quad-3 c+5 d+4=1 \quad \Rightarrow-3 c+5 d=-3 \tag{7} \end{equation*} $$

and from eq. (3)

$$ \begin{align*} 4 \times 1+2 c-6 \times 1-3 d & =0 ; \\ \Rightarrow \quad & \Rightarrow 4+2 c-6-3 d=0 \tag{8}\\ -2+2 c-3 d & =0 \quad \Rightarrow \quad 2 c-3 d=2 \end{align*} $$

Solving eq. (7) and (8) we get,

$ \begin{matrix} 2 \times(-3 c+5 d=-3) & \Rightarrow & -6 c+10 d=-6 \\ 3 \times(2 c-3 d=2) & \Rightarrow & \text{ Adding } \end{matrix} $

Putting the value of $d$ in eq. (8) we get,

$ 2 c-3 \times 0=2 \Rightarrow 2 c=2 \Rightarrow c=1 $

Hence, $A= \begin{bmatrix} 1 & 1 \\ 1 & 0 \end{bmatrix} $.

13. Find $A$, if $ \begin{bmatrix} 4 \\ 1 \\ 3\end{bmatrix} A= \begin{bmatrix} -4 & 8 & 4 \\ -1 & 2 & 1 \\ -3 & 6 & 3 \end{bmatrix} $

Show Answer

Solution

Order of $ \begin{bmatrix} 4 \\ 1 \\ 3\end{bmatrix} $ is $3 \times 1$ and order of $ \begin{bmatrix} -4 & 8 & 4 \\ -1 & 2 & 1 \\ -3 & 6 & 3 \end{bmatrix} $ is $3 \times 3$.

So, the order of matrix A must be $1 \times 3$.

Let $A= \begin{bmatrix} a & b & c \end{bmatrix} _{1 \times 3}$

$ \begin{aligned} { \begin{bmatrix} 4 \\ 1 \\ 3 \end{bmatrix} _{3 \times 1} \begin{bmatrix} a & b & c \end{bmatrix} _{1 \times 3} } & = \begin{bmatrix} -4 & 8 & 4 \\ -1 & 2 & 1 \\ -3 & 6 & 3 \end{bmatrix} _{3 \times 3} \\ \Rightarrow \quad \begin{bmatrix} 4 a & 4 b & 4 c \\ a & b & c \\ 3 a & 3 b & 3 c \end{bmatrix} & = \begin{bmatrix} -4 & 8 & 4 \\ -1 & 2 & 1 \\ -3 & 6 & 3 \end{bmatrix} \end{aligned} $

Equating the corresponding elements to get the values of $a, b$ and $c$

$ \begin{aligned} & 4 a=-4, \quad 4 b=8, \quad 4 c=4 \\ & \therefore \quad a=-1 \quad \therefore \quad b=2 \quad \therefore c=1 \end{aligned} $

14. If $A= \begin{bmatrix} 3 & -4 \\ 1 & 1 \\ 2 & 0\end{bmatrix} $ and $B= \begin{bmatrix} 2 & 1 & 2 \\ 1 & 2 & 4 \end{bmatrix} $, then verify $(B A)^{2} \neq B^{2} A^{2}$.

Show Answer

Solution

Here, $\quad B= \begin{bmatrix} 2 & 1 & 2 \\ 1 & 2 & 4\end{bmatrix} _{2 \times 3}$ and $A= \begin{bmatrix} 3 & -4 \\ 1 & 1 \\ 2 & 0 \end{bmatrix} _{3 \times 2}$

$\therefore \quad BA= \begin{bmatrix} 6+1+4 & -8+1+0 \\ 3+2+8 & -4+2+0\end{bmatrix} _{2 \times 2} \Rightarrow BA= \begin{bmatrix} 11 & -7 \\ 13 & -2 \end{bmatrix} $

L.H.S. $\quad(BA)^{2}=(BA) \cdot(BA)= \begin{bmatrix} 11 & -7 \\ 13 & -2\end{bmatrix} \begin{bmatrix} 11 & -7 \\ 13 & -2 \end{bmatrix} $

$\Rightarrow \quad \begin{bmatrix} 121-91 & -77+14 \\ 143-26 & -91+4\end{bmatrix} \Rightarrow \begin{bmatrix} 30 & -63 \\ 117 & -87 \end{bmatrix} $

R.H.S. $B^{2}=B \cdot B= \begin{bmatrix} 2 & 1 & 2 \\ 1 & 2 & 4\end{bmatrix} _{2 \times 3} \cdot \begin{bmatrix} 2 & 1 & 2 \\ 1 & 2 & 4 \end{bmatrix} _{2 \times 3}$

Here, number of columns of first i.e., 3 is not equal to the number of rows of second matrix i.e., 2.

So, $B^{2}$ is not possible. Similarly, $A^{2}$ is also not possible.

Hence, $(BA)^{2} \neq B^{2} A^{2}$

15. If possible, find $B A$ and $A B$, where

$ A= \begin{bmatrix} 2 & 1 & 2 \\ 1 & 2 & 4 \end{bmatrix} , B= \begin{bmatrix} 4 & 1 \\ 2 & 3 \\ 1 & 2 \end{bmatrix} $

Show Answer

Solution

$\quad \begin{aligned} B A & = \begin{bmatrix} 4 & 1 \\ 2 & 3 \\ 1 & 2\end{bmatrix} _{3 \times 2} \begin{bmatrix} 2 & 1 & 2 \\ 1 & 2 & 4\end{bmatrix} _{2 \times 3} \\ B A & = \begin{bmatrix} 8+1 & 4+2 & 8+4 \\ 4+3 & 2+6 & 4+12 \\ 2+2 & 1+4 & 2+8\end{bmatrix} _{3 \times 3}= \begin{bmatrix} 9 & 6 & 12 \\ 7 & 8 & 16 \\ 4 & 5 & 10 \end{bmatrix} _{3 \times 3}\end{aligned}$

Now $A B= \begin{bmatrix} 2 & 1 & 2 \\ 1 & 2 & 4\end{bmatrix} _{2 \times 3} \begin{bmatrix} 4 & 1 \\ 2 & 3 \\ 1 & 2 \end{bmatrix} _{3 \times 2}$

$ = \begin{bmatrix} 8+2+2 & 2+3+4 \\ 4+4+4 & 1+6+8 \end{bmatrix} _{2 \times 2}= \begin{bmatrix} 12 & 9 \\ 12 & 15 \end{bmatrix} _{2 \times 2} $

Hence, $\quad BA= \begin{bmatrix} 9 & 6 & 12 \\ 7 & 8 & 16 \\ 4 & 5 & 10\end{bmatrix} $ and $AB= \begin{bmatrix} 12 & 9 \\ 12 & 15 \end{bmatrix} $.

16. Show by an example that for $A \neq O$ and $B \neq O, A B=O$.

Show Answer

Solution

Let $A= \begin{bmatrix} 1 & -1 \\ -1 & 1\end{bmatrix} $ and $B= \begin{bmatrix} 1 & 1 \\ 1 & 1 \end{bmatrix} $

$ \begin{aligned} & AB= \begin{bmatrix} 1 & -1 \\ -1 & 1 \end{bmatrix} \begin{bmatrix} 1 & 1 \\ 1 & 1 \end{bmatrix} \\ & \Rightarrow \quad AB= \begin{bmatrix} 1-1 & 1-1 \\ -1+1 & -1+1 \end{bmatrix} = \begin{bmatrix} 0 & 0 \\ 0 & 0 \end{bmatrix} =O \end{aligned} $

Hence, $\quad A= \begin{bmatrix} 1 & -1 \\ -1 & 1\end{bmatrix} $ and $B= \begin{bmatrix} 1 & 1 \\ 1 & 1 \end{bmatrix} $.

17. Given $A= \begin{bmatrix} 2 & 4 & 0 \\ 3 & 9 & 6\end{bmatrix} $ and $B= \begin{bmatrix} 1 & 4 \\ 2 & 8 \\ 1 & 3 \end{bmatrix} $. Is $(AB)^{\prime}=B^{\prime} A^{\prime}$ ?

Show Answer

Solution

Here, $A= \begin{bmatrix} 2 & 4 & 0 \\ 3 & 9 & 6\end{bmatrix} , B= \begin{bmatrix} 1 & 4 \\ 2 & 8 \\ 1 & 3 \end{bmatrix} $

$ \begin{aligned} AB & = \begin{bmatrix} 2 & 4 & 0 \\ 3 & 9 & 6 \end{bmatrix} \begin{bmatrix} 1 & 4 \\ 2 & 8 \\ 1 & 3 \end{bmatrix} \\ & = \begin{bmatrix} 2+8+0 & 8+32+0 \\ 3+18+6 & 12+72+18 \end{bmatrix} = \begin{bmatrix} 10 & 40 \\ 27 & 102 \end{bmatrix} \end{aligned} $

L.H.S. $(A B)^{\prime}= \begin{bmatrix} 10 & 27 \\ 40 & 102 \end{bmatrix} $

Now $B= \begin{bmatrix} 1 & 4 \\ 2 & 8 \\ 1 & 3\end{bmatrix} \quad \Rightarrow \quad B^{\prime}= \begin{bmatrix} 1 & 2 & 1 \\ 4 & 8 & 3 \end{bmatrix} $

$ A= \begin{bmatrix} 2 & 4 & 0 \\ 3 & 9 & 6 \end{bmatrix} \Rightarrow A^{\prime}= \begin{bmatrix} 2 & 3 \\ 4 & 9 \\ 0 & 6 \end{bmatrix} $

R.H.S. $B^{\prime} A^{\prime}= \begin{bmatrix} 1 & 2 & 1 \\ 4 & 8 & 3\end{bmatrix} \begin{bmatrix} 2 & 3 \\ 4 & 9 \\ 0 & 6 \end{bmatrix} $

$ = \begin{bmatrix} 2+8+0 & 3+18+6 \\ 8+32+0 & 12+72+18 \end{bmatrix} = \begin{bmatrix} 10 & 27 \\ 40 & 102 \end{bmatrix} =\text{ L.H.S. } $

Hence, L.H.S. = R.H.S.

18. Solve for $x$ and $y: x \begin{bmatrix} 2 \\ 1\end{bmatrix} +y \begin{bmatrix} 3 \\ 5\end{bmatrix} + \begin{bmatrix} -8 \\ -11 \end{bmatrix} =O$

Show Answer

Solution

Given that: $x \begin{bmatrix} 2 \\ 1\end{bmatrix} +y \begin{bmatrix} 3 \\ 5\end{bmatrix} + \begin{bmatrix} -8 \\ 11 \end{bmatrix} =$

L.H.S. $x \begin{bmatrix} 2 \\ 1\end{bmatrix} +y \begin{bmatrix} 3 \\ 5\end{bmatrix} + \begin{bmatrix} -8 \\ -11 \end{bmatrix} =0$

$\Rightarrow \quad \begin{bmatrix} 2 x \\ x\end{bmatrix} + \begin{bmatrix} 3 y \\ 5 y\end{bmatrix} + \begin{bmatrix} -8 \\ -11\end{bmatrix} =O \Rightarrow \begin{bmatrix} 2 x+3 y-8 \\ x+5 y-11\end{bmatrix} = \begin{bmatrix} 0 \\ 0 \end{bmatrix} $

Comparing the corresponding elements of both sides, we get,

$ \begin{aligned} & 2 x+3 y-8=0 \quad \Rightarrow \quad 2 x+3 y=8 \\ & x+5 y-11=0 \quad \Rightarrow \quad x+5 y=11 \end{aligned} $

Multiplying eq. (1) by 1 and eq. (2) by 2, and then on subtracting, we get,

$$ \begin{aligned} 2 x+3 y=8 \\ 2 x+10 y=22 \\ (-) \quad(-) \quad(-) \\ \hline -7y =-14 \end{aligned} $$

$\therefore \quad y=2$

Putting $y=2$ in eq. (2) we get,

$ x+5 \times 2=11 \Rightarrow x+10=11 $

$ \therefore \quad x=11-10=1 $

Hence, the values of $x$ and $y$ are 1 and 2 respectively.

19. If $X$ and $Y$ are $2 \times 2$ matrices, then solve the following matrix equations for $X$ and $Y$.

$ 2 X+3 Y= \begin{bmatrix} 2 & 3 \\ 4 & 0 \end{bmatrix} , 3 X+2 Y= \begin{bmatrix} -2 & 2 \\ 1 & -5 \end{bmatrix} $

Show Answer

Solution

Given that:

$$ \begin{align*} & 2 X+3 Y= \begin{bmatrix} 2 & 3 \\ 4 & 0 \end{bmatrix} \tag{1}\\ & 3 X+2 Y= \begin{bmatrix} -2 & 2 \\ 1 & -5 \end{bmatrix} \tag{2} \end{align*} $$

Multiplying eq. (1) by 3 and eq. (2) by 2, we get,

$ \begin{aligned} & 3[2 X+3 Y]=3 \begin{bmatrix} 2 & 3 \\ 4 & 0 \end{bmatrix} \quad \Rightarrow \quad 6 X+9 Y= \begin{bmatrix} 6 & 9 \\ 12 & 0 \end{bmatrix} \\ & 2[3 X+2 Y]=2 \begin{bmatrix} -2 & 2 \\ 1 & -5 \end{bmatrix} \Rightarrow 6 X+4 Y= \begin{bmatrix} -4 & 4 \\ 2 & -10 \end{bmatrix} \end{aligned} $

On subtracting eq. (4) from eq. (3) we get

$ \begin{aligned} & 5 Y= \begin{bmatrix} 6+4 & 9-4 \\ 12-2 & 0+10 \end{bmatrix} \\ & 5 Y= \begin{bmatrix} 10 & 5 \\ 10 & 10 \end{bmatrix} \Rightarrow Y= \begin{bmatrix} 2 & 1 \\ 2 & 2 \end{bmatrix} \end{aligned} $

Now, putting the value of $Y$ in equation (1) we get,

$ \begin{aligned} & 2 X+3 \begin{bmatrix} 2 & 1 \\ 2 & 2 \end{bmatrix} = \begin{bmatrix} 2 & 3 \\ 4 & 0 \end{bmatrix} \\ & \Rightarrow 2 X+ \begin{bmatrix} 6 & 3 \\ 6 & 6 \end{bmatrix} = \begin{bmatrix} 2 & 3 \\ 4 & 0 \end{bmatrix} \quad \Rightarrow 2 X= \begin{bmatrix} 2 & 3 \\ 4 & 0 \end{bmatrix} - \begin{bmatrix} 6 & 3 \\ 6 & 6 \end{bmatrix} \\ & \Rightarrow \quad 2 X= \begin{bmatrix} 2-6 & 3-3 \\ 4-6 & 0-6 \end{bmatrix} \Rightarrow 2 X= \begin{bmatrix} -4 & 0 \\ -2 & -6 \end{bmatrix} \\ & \Rightarrow \quad X=\frac{1}{2} \begin{bmatrix} -4 & 0 \\ -2 & -6 \end{bmatrix} \quad \Rightarrow \quad X= \begin{bmatrix} -2 & 0 \\ -1 & -3 \end{bmatrix} \end{aligned} $

Hence, $X= \begin{bmatrix} -2 & 0 \\ -1 & -3\end{bmatrix} $ and $Y= \begin{bmatrix} 2 & 1 \\ 2 & 2 \end{bmatrix} $.

20. If $A= \begin{bmatrix} 3 & 5\end{bmatrix} , B= \begin{bmatrix} 7 & 3 \end{bmatrix} $, then find a non-zero matrix $C$ such that $AC=BC$.

Show Answer

Solution

Given that: $A= \begin{bmatrix} 3 & 5\end{bmatrix} _{1 \times 2}, B= \begin{bmatrix} 7 & 3 \end{bmatrix} _{1 \times 2}$

Let

$ \begin{aligned} C & = \begin{bmatrix} \alpha \\ \beta \end{bmatrix} _{2 \times 1} \\ AC & = \begin{bmatrix} 3 & 5 \end{bmatrix} _{1 \times 2} \begin{bmatrix} \alpha \\ \beta \end{bmatrix} _{2 \times 1}=[3 \alpha+5 \beta] \\ BC & = \begin{bmatrix} 7 & 3 \end{bmatrix} _{1 \times 2} \begin{bmatrix} \alpha \\ \beta \end{bmatrix} _{2 \times 1}=[7 \alpha+3 \beta] \\ AC & =BC \end{aligned} $

$ \begin{matrix} \Rightarrow & {[3 \alpha+5 \beta]=[7 \alpha+3 \beta]} \\ \Rightarrow & 3 \alpha+5 \beta=7 \alpha+3 \beta \end{matrix} $

$ \begin{matrix} \Rightarrow & 3 \alpha-7 \alpha & =3 \beta-5 \beta \\ \Rightarrow & -4 \alpha & =-2 \beta \\ & \therefore & \frac{\alpha}{\beta} & =\frac{1}{2} \end{matrix} $

So, let $\alpha=K$ and $\beta=2 K$, $K$ is some real number.

Hence, matrix $C= \begin{bmatrix} K \\ 2 K\end{bmatrix} _{2 \times 1}$ or $ \begin{bmatrix} K & K \\ 2 K & 2 K \end{bmatrix} _{2 \times 2}$ etc.

21. Give an example of matrices $A, B$ and $C$ such that $A B=A C$, where $A$ is non-zero matrix, but $B \neq C$.

Show Answer

Solution

Let $A= \begin{bmatrix} 1 & 0 \\ 0 & 0\end{bmatrix} , B= \begin{bmatrix} 1 & 2 \\ 2 & 0\end{bmatrix} $ and $C= \begin{bmatrix} 1 & 2 \\ 2 & 2 \end{bmatrix} $

$ \begin{aligned} & AB= \begin{bmatrix} 1 & 0 \\ 0 & 0 \end{bmatrix} \begin{bmatrix} 1 & 2 \\ 2 & 0 \end{bmatrix} \Rightarrow AB= \begin{bmatrix} 1+0 & 2+0 \\ 0+0 & 0+0 \end{bmatrix} = \begin{bmatrix} 1 & 2 \\ 0 & 0 \end{bmatrix} \\ & AC= \begin{bmatrix} 1 & 0 \\ 0 & 0 \end{bmatrix} \begin{bmatrix} 1 & 2 \\ 2 & 2 \end{bmatrix} = \begin{bmatrix} 1+0 & 2+0 \\ 0+0 & 0+0 \end{bmatrix} = \begin{bmatrix} 1 & 2 \\ 0 & 0 \end{bmatrix} \end{aligned} $

Hence, $A B=A C$ for matrix $A$ is non-zero and $B \neq C$.

22. If $A= \begin{bmatrix} 1 & 2 \\ -2 & 1\end{bmatrix} , B= \begin{bmatrix} 2 & 3 \\ 3 & -4\end{bmatrix} $ and $C= \begin{bmatrix} 1 & 0 \\ -1 & 0 \end{bmatrix} $

verify: (i) $(AB) C=A(BC) \quad$ (ii) $A(B+C)=AB+AC$

Show Answer

Solution

Given that $A= \begin{bmatrix} 1 & 2 \\ -2 & 1\end{bmatrix} , B= \begin{bmatrix} 2 & 3 \\ 3 & -4\end{bmatrix} $ and $C= \begin{bmatrix} 1 & 0 \\ -1 & 0 \end{bmatrix} $

(i) To verify: $(AB) C=A(BC)$

$ AB= \begin{bmatrix} 1 & 2 \\ -2 & 1 \end{bmatrix} \begin{bmatrix} 2 & 3 \\ 3 & -4 \end{bmatrix} = \begin{bmatrix} 2+6 & 3-8 \\ -4+3 & -6-4 \end{bmatrix} = \begin{bmatrix} 8 & -5 \\ -1 & -10 \end{bmatrix} $

L.H.S.

$ \begin{aligned} (A B) C & = \begin{bmatrix} 8 & -5 \\ -1 & -10 \end{bmatrix} \begin{bmatrix} 1 & 0 \\ -1 & 0 \end{bmatrix} = \begin{bmatrix} 8+5 & 0+0 \\ -1+10 & 0+0 \end{bmatrix} = \begin{bmatrix} 13 & 0 \\ 9 & 0 \end{bmatrix} \\ BC & = \begin{bmatrix} 2 & 3 \\ 3 & -4 \end{bmatrix} \begin{bmatrix} 1 & 0 \\ -1 & 0 \end{bmatrix} = \begin{bmatrix} 2-3 & 0+0 \\ 3+4 & 0+0 \end{bmatrix} = \begin{bmatrix} -1 & 0 \\ 7 & 0 \end{bmatrix} \end{aligned} $

R.H.S.

$ A(BC)= \begin{bmatrix} 1 & 2 \\ -2 & 1 \end{bmatrix} \begin{bmatrix} -1 & 0 \\ 7 & 0 \end{bmatrix} = \begin{bmatrix} -1+14 & 0+0 \\ 2+7 & 0+0 \end{bmatrix} = \begin{bmatrix} 13 & 0 \\ 9 & 0 \end{bmatrix} $

L.H.S. $=$ R.H.S.

So, $(AB) C=A(BC)$

(ii) To verify: $A(B+C)=AB+AC$

L.H.S. $B+C= \begin{bmatrix} 2 & 3 \\ 3 & -4\end{bmatrix} + \begin{bmatrix} 1 & 0 \\ -1 & 0 \end{bmatrix} =$

$ = \begin{bmatrix} 2+1 & 3+0 \\ 3-1 & -4+0 \end{bmatrix} = \begin{bmatrix} 3 & 3 \\ 2 & -4 \end{bmatrix} $

L.H.S.A $(B+C)= \begin{bmatrix} 1 & 2 \\ -2 & 1\end{bmatrix} \begin{bmatrix} 3 & 3 \\ 2 & -4 \end{bmatrix} $

$ = \begin{bmatrix} 3+4 & 3-8 \\ -6+2 & -6-4 \end{bmatrix} = \begin{bmatrix} 7 & -5 \\ -4 & -10 \end{bmatrix} $

R.H.S. $AB= \begin{bmatrix} 1 & 2 \\ -2 & 1\end{bmatrix} \begin{bmatrix} 2 & 3 \\ 3 & -4 \end{bmatrix} $

$ = \begin{bmatrix} 2+6 & 3-8 \\ -4+3 & -6-4 \end{bmatrix} = \begin{bmatrix} 8 & -5 \\ -1 & -10 \end{bmatrix} $

$ \begin{aligned} AC & = \begin{bmatrix} 1 & 2 \\ -2 & 1 \end{bmatrix} \begin{bmatrix} 1 & 0 \\ -1 & 0 \end{bmatrix} \\ & = \begin{bmatrix} 1-2 & 0+0 \\ -2-1 & 0+0 \end{bmatrix} = \begin{bmatrix} -1 & 0 \\ -3 & 0 \end{bmatrix} \end{aligned} $

R.H.S. $AB+AC= \begin{bmatrix} 8 & -5 \\ -1 & -10\end{bmatrix} + \begin{bmatrix} -1 & 0 \\ -3 & 0\end{bmatrix} = \begin{bmatrix} 8-1 & -5+0 \\ -1-3 & -10+0 \end{bmatrix} $

$ = \begin{bmatrix} 7 & -5 \\ -4 & -10 \end{bmatrix} $

L.H.S. $=$ R.H.S.

Hence, $A(B+C)=AB+AC$

23. If $P= \begin{bmatrix} x & 0 & 0 \\ 0 & y & 0 \\ 0 & 0 & z\end{bmatrix} $ and $Q= \begin{bmatrix} a & 0 & 0 \\ 0 & b & 0 \\ 0 & 0 & c \end{bmatrix} $, prove that

$ PQ= \begin{bmatrix} x a & 0 & 0 \\ 0 & y b & 0 \\ 0 & 0 & z c \end{bmatrix} =QP $

Show Answer

Solution

Given that: $P= \begin{bmatrix} x & 0 & 0 \\ 0 & y & 0 \\ 0 & 0 & z\end{bmatrix} $ and $Q= \begin{bmatrix} a & 0 & 0 \\ 0 & b & 0 \\ 0 & 0 & c \end{bmatrix} $

$PQ= \begin{bmatrix} x & 0 & 0 \\ 0 & y & 0 \\ 0 & 0 & z\end{bmatrix} \begin{bmatrix} a & 0 & 0 \\ 0 & b & 0 \\ 0 & 0 & c \end{bmatrix} $

$PQ= \begin{bmatrix} x a+0+0 & 0+0+0 & 0+0+0 \\ 0+0+0 & 0+y b+0 & 0+0+0 \\ 0+0+0 & 0+0+0 & 0+0+z c \end{bmatrix} $

$PQ= \begin{bmatrix} x a & 0 & 0 \\ 0 & y b & 0 \\ 0 & 0 & z c \end{bmatrix} $

Now QP $= \begin{bmatrix} a & 0 & 0 \\ 0 & b & 0 \\ 0 & 0 & c\end{bmatrix} \begin{bmatrix} x & 0 & 0 \\ 0 & y & 0 \\ 0 & 0 & z \end{bmatrix} $

$QP= \begin{bmatrix} x a+0+0 & 0+0+0 & 0+0+0 \\ 0+0+0 & 0+y b+0 & 0+0+0 \\ 0+0+0 & 0+0+0 & 0+0+z c \end{bmatrix} $

$QP= \begin{bmatrix} x a & 0 & 0 \\ 0 & y b & 0 \\ 0 & 0 & z c \end{bmatrix} $

Hence, $PQ=QP$.

24. If $ \begin{bmatrix} 2 & 1 & 3\end{bmatrix} \begin{bmatrix} -1 & 0 & -1 \\ -1 & 1 & 0 \\ 0 & 1 & 1\end{bmatrix} \begin{bmatrix} 1 \\ 0 \\ -1 \end{bmatrix} =A$ find $A$.

Show Answer

Solution

Given that: $ \begin{bmatrix} 2 & 1 & 3\end{bmatrix} \begin{bmatrix} -1 & 0 & -1 \\ -1 & 1 & 0 \\ 0 & 1 & 1\end{bmatrix} \begin{bmatrix} 1 \\ 0 \\ -1 \end{bmatrix} =A$

L.H.S. $\quad \begin{bmatrix} 2 & 1 & 3\end{bmatrix} _{1 \times 3} \begin{bmatrix} -1 & 0 & -1 \\ -1 & 1 & 0 \\ 0 & 1 & 1\end{bmatrix} _{3 \times 3} \begin{bmatrix} 1 \\ 0 \\ -1 \end{bmatrix} _{3 \times 1}$

$ \begin{matrix} \Rightarrow & { \begin{bmatrix} -2-1+0 & 0+1+3 & -2+0+3 \end{bmatrix} _{1 \times 3} \begin{bmatrix} 1 \\ 0 \\ -1 \end{bmatrix} _{3 \times 1}} \\ \Rightarrow & { \begin{bmatrix} -3 & 4 & 1 \end{bmatrix} _{1 \times 3} \begin{bmatrix} 1 \\ 0 \\ -1 \end{bmatrix} _{3 \times 1}} \\ \Rightarrow \quad & {[-3+0-1] _{1 \times 1}=[-4] _{1 \times 1}} \end{matrix} $

Hence, matrix $A=[-4]$

25. If $A= \begin{bmatrix} 2 & 1\end{bmatrix} , B= \begin{bmatrix} 5 & 3 & 4 \\ 8 & 7 & 6\end{bmatrix} $ and $C= \begin{bmatrix} -1 & 2 & 1 \\ 1 & 0 & 2 \end{bmatrix} $,

verify that $A(B+C)=(A B+A C)$.

Show Answer

Solution

Given that: $A= \begin{bmatrix} 2 & 1\end{bmatrix} , B= \begin{bmatrix} 5 & 3 & 4 \\ 8 & 7 & 6\end{bmatrix} $ and $C= \begin{bmatrix} -1 & 2 & 1 \\ 1 & 0 & 2 \end{bmatrix} $.

L.H.S. $\quad(B+C)= \begin{bmatrix} 5 & 3 & 4 \\ 8 & 7 & 6\end{bmatrix} + \begin{bmatrix} -1 & 2 & 1 \\ 1 & 0 & 2 \end{bmatrix} $

$ = \begin{bmatrix} 5-1 & 3+2 & 4+1 \\ 8+1 & 7+0 & 2+6 \end{bmatrix} = \begin{bmatrix} 4 & 5 & 5 \\ 9 & 7 & 8 \end{bmatrix} $

$ \begin{aligned} A(B+C) & = \begin{bmatrix} 2 & 1 \end{bmatrix} _{1 \times 2} \begin{bmatrix} 4 & 5 & 5 \\ 9 & 7 & 8 \end{bmatrix} _{2 \times 3} \\ & = \begin{bmatrix} 8+9 & 10+7 & 10+8 \end{bmatrix} _{1 \times 3} \\ A(B+C) & = \begin{bmatrix} 17 & 17 & 18 \end{bmatrix} \end{aligned} $

R.H.S. $A B= \begin{bmatrix} 2 & 1\end{bmatrix} _{1 \times 2} \begin{bmatrix} 5 & 3 & 4 \\ 8 & 7 & 6 \end{bmatrix} _{2 \times 3}$

$ \begin{aligned} & = \begin{bmatrix} 10+8 & 6+7 & 8+6 \end{bmatrix} _{1 \times 3}= \begin{bmatrix} 18 & 13 & 14 \end{bmatrix} _{1 \times 3} \\ & AC= \begin{bmatrix} 2 & 1 \end{bmatrix} _{1 \times 2} \begin{bmatrix} -1 & 2 & 1 \\ 1 & 0 & 2 \end{bmatrix} _{2 \times 3} \\ & = \begin{bmatrix} -2+1 & 4+0 & 2+2 \end{bmatrix} _{1 \times 3}= \begin{bmatrix} -1 & 4 & 4 \end{bmatrix} _{1 \times 3} \\ & AB+AC= \begin{bmatrix} 18 & 13 & 14 \end{bmatrix} _{1 \times 3}+ \begin{bmatrix} -1 & 4 & 4 \end{bmatrix} _{1 \times 3} \\ & = \begin{bmatrix} 18-1 & 13+4 & 14+4 \end{bmatrix} _{1 \times 3} \\ & AB+AC= \begin{bmatrix} 17 & 17 & 18 \end{bmatrix} _{1 \times 3} \\ & \text{ L.H.S. = R.H.S. } \end{aligned} $

26. If $A= \begin{bmatrix} 1 & 0 & -1 \\ 2 & 1 & 3 \\ 0 & 1 & 1 \end{bmatrix} $ then verify that $A^{2}+A=A(A+I)$, where $I$ is $3 \times 3$ unit matrix.

Show Answer

Solution

Given that: $A= \begin{bmatrix} 1 & 0 & -1 \\ 2 & 1 & 3 \\ 0 & 1 & 1 \end{bmatrix} $

$ \begin{aligned} A^{2} & =A \cdot A \\ & = \begin{bmatrix} 1 & 0 & -1 \\ 2 & 1 & 3 \\ 0 & 1 & 1 \end{bmatrix} \begin{bmatrix} 1 & 0 & -1 \\ 2 & 1 & 3 \\ 0 & 1 & 1 \end{bmatrix} \\ & = \begin{bmatrix} 1+0+0 & 0+0-1 & -1+0-1 \\ 2+2+0 & 0+1+3 & -2+3+3 \\ 0+2+0 & 0+1+1 & 0+3+1 \end{bmatrix} = \begin{bmatrix} 1 & -1 & -2 \\ 4 & 4 & 4 \\ 2 & 2 & 4 \end{bmatrix} \end{aligned} $

L.H.S. $\quad A^{2}+A= \begin{bmatrix} 1 & -1 & -2 \\ 4 & 4 & 4 \\ 2 & 2 & 4\end{bmatrix} \begin{bmatrix} 1 & 0 & -1 \\ 2 & 1 & 3 \\ 0 & 1 & 1 \end{bmatrix} $

$= \begin{bmatrix} 1+1 & -1+0 & -2-1 \\ 4+2 & 4+1 & 4+3 \\ 2+0 & 2+1 & 4+1\end{bmatrix} = \begin{bmatrix} 2 & -1 & -3 \\ 6 & 5 & 7 \\ 2 & 3 & 5 \end{bmatrix} $

R.H.S. $A(A+I)= \begin{bmatrix} 1 & 0 & -1 \\ 2 & 1 & 3 \\ 0 & 1 & 1 \end{bmatrix} [(\begin{matrix} 1 & 0 & -1 \\ 2 & 1 & 3 \\ 0 & 1 & 1\end{matrix} )+(\begin{matrix} 1 & 0 & 0 \\ 0 & 1 & 0 \\ 0 & 0 & 1\end{matrix} )]$

$= \begin{bmatrix} 1 & 0 & -1 \\ 2 & 1 & 3 \\ 0 & 1 & 1\end{bmatrix} \begin{bmatrix} 2 & 0 & -1 \\ 2 & 2 & 3 \\ 0 & 1 & 2 \end{bmatrix} $

$= \begin{bmatrix} 2+0+0 & 0+0-1 & -1+0-2 \\ 4+2+0 & 0+2+3 & -2+3+6 \\ 0+2+0 & 0+2+1 & 0+3+2\end{bmatrix} = \begin{bmatrix} 2 & -1 & -3 \\ 6 & 5 & 7 \\ 2 & 3 & 5 \end{bmatrix} $

L.H.S. $=$ R.H.S.

$A^{2}+A=A(A+I)$. Hence verified.

27. If $A= \begin{bmatrix} 0 & -1 & 2 \\ 4 & 3 & -4\end{bmatrix} $ and $B= \begin{bmatrix} 4 & 0 \\ 1 & 3 \\ 2 & 6 \end{bmatrix} $, then verify that:

(i) $(A^{\prime})^{\prime}=A$

(ii) $(AB)^{\prime}=B^{\prime} A^{\prime}$

(iii) $(k A)^{\prime}=(k A^{\prime})$

Show Answer

Solution

Given that: $A= \begin{bmatrix} 0 & -1 & 2 \\ 4 & 3 & -4\end{bmatrix} , B= \begin{bmatrix} 4 & 0 \\ 1 & 3 \\ 2 & 6 \end{bmatrix} $

(i)

$ \begin{aligned} A^{\prime} & = \begin{bmatrix} 0 & -1 & 2 \\ 4 & 3 & -4 \end{bmatrix} _{2 \times 3}^{\prime}= \begin{bmatrix} 0 & 4 \\ -1 & 3 \\ 2 & -4 \end{bmatrix} _{3 \times 2} \\ (A^{\prime})^{\prime} & = \begin{bmatrix} 0 & 4 \\ -1 & 3 \\ 2 & -4 \end{bmatrix} _{3 \times 2}^{\prime}= \begin{bmatrix} 0 & -1 & 2 \\ 4 & 3 & -4 \end{bmatrix} _{2 \times 3}=A \end{aligned} $

Hence, $(A^{\prime})^{\prime}=A$

(ii) L.H.S. $\quad AB= \begin{bmatrix} 0 & -1 & 2 \\ 4 & 3 & -4\end{bmatrix} _{2 \times 3} \begin{bmatrix} 4 & 0 \\ 1 & 3 \\ 2 & 6 \end{bmatrix} _{3 \times 2}$

$= \begin{bmatrix} 0-1+4 & 0-3+12 \\ 16+3-8 & 0+9-24\end{bmatrix} _{2 \times 2}= \begin{bmatrix} 3 & 9 \\ 11 & -15 \end{bmatrix} _{2 \times 2}$

$(A B)^{\prime}= \begin{bmatrix} 3 & 11 \\ 9 & -15 \end{bmatrix} _{2 \times 2}$

R.H.S. $B^{\prime}= \begin{bmatrix} 4 & 0 \\ 1 & 3 \\ 2 & 6\end{bmatrix} ^{\prime}= \begin{bmatrix} 4 & 1 & 2 \\ 0 & 3 & 6 \end{bmatrix} $

$A^{\prime}= \begin{bmatrix} 0 & -1 & 2 \\ 4 & 3 & -4\end{bmatrix} ^{\prime}= \begin{bmatrix} 0 & 4 \\ -1 & 3 \\ 2 & -4 \end{bmatrix} $

$B^{\prime} A^{\prime}= \begin{bmatrix} 4 & 1 & 2 \\ 0 & 3 & 6\end{bmatrix} _{2 \times 3} \begin{bmatrix} 0 & 4 \\ -1 & 3 \\ 2 & -4 \end{bmatrix} _{3 \times 2}$

$= \begin{bmatrix} 0-1+4 & 16+3-8 \\ 0-3+12 & 0+9-24\end{bmatrix} _{2 \times 2}= \begin{bmatrix} 3 & 11 \\ 9 & -15 \end{bmatrix} _{2 \times 2}$

L.H.S. $=$ R.H.S.

Hence, $(A B)^{\prime}=B^{\prime} A^{\prime}$ is verified.

(iii) L.H.S. $\quad k A=k \begin{bmatrix} 0 & -1 & 2 \\ 4 & 3 & -4\end{bmatrix} = \begin{bmatrix} 0 & -k & 2 k \\ 4 k & 3 k & -4 k \end{bmatrix} $

$ (k A)^{\prime}= \begin{bmatrix} 0 & 4 k \\ -k & 3 k \\ 2 k & -4 k \end{bmatrix} $

R.H.S. $k A^{\prime}=k \begin{bmatrix} 0 & -1 & 2 \\ 4 & 3 & -4\end{bmatrix} ^{\prime}=k \begin{bmatrix} 0 & 4 \\ -1 & 3 \\ 2 & -4\end{bmatrix} = \begin{bmatrix} 0 & 4 k \\ -k & 3 k \\ 2 k & -4 k \end{bmatrix} $

Hence, L.H.S. $=$ R.H.S.

$(k A)^{\prime}=(k A^{\prime})$ is verified.

28. If $A= \begin{bmatrix} 1 & 2 \\ 4 & 1 \\ 5 & 6\end{bmatrix} , B= \begin{bmatrix} 1 & 2 \\ 6 & 4 \\ 7 & 3 \end{bmatrix} $, then verify that:

(i) $(2 A+B)^{\prime}=2 A^{\prime}+B^{\prime}$

(ii) $(A-B)^{\prime}=A^{\prime}-B^{\prime}$

Show Answer

Solution

Given that: $A= \begin{bmatrix} 1 & 2 \\ 4 & 1 \\ 5 & 6\end{bmatrix} $ and $B= \begin{bmatrix} 1 & 2 \\ 6 & 4 \\ 7 & 3 \end{bmatrix} $

(i) To verify that: $(2 A+B)^{\prime}=2 A^{\prime}+B^{\prime}$

$ \begin{aligned} & \text{ L.H.S. }(2 A+B)^{\prime}=[2(\begin{matrix} 1 & 2 \\ 4 & 1 \\ 5 & 6 \end{matrix} )+(\begin{matrix} 1 & 2 \\ 6 & 4 \\ 7 & 3 \end{matrix} )]^{\prime}=[(\begin{matrix} 2 & 4 \\ 8 & 2 \\ 10 & 12 \end{matrix} )+(\begin{matrix} 1 & 2 \\ 6 & 4 \\ 7 & 3 \end{matrix} )]^{\prime} \\ & = \begin{bmatrix} 2+1 & 4+2 \\ 8+6 & 2+4 \\ 10+7 & 12+3 \end{bmatrix} ^{\prime}= \begin{bmatrix} 3 & 6 \\ 14 & 6 \\ 17 & 15 \end{bmatrix} ^{\prime}= \begin{bmatrix} 3 & 14 & 17 \\ 6 & 6 & 15 \end{bmatrix} \\ & \text{ R.H.S. } 2 A^{\prime}+B^{\prime}=2 \begin{bmatrix} 1 & 2 \\ 4 & 1 \\ 5 & 6 \end{bmatrix} ^{\prime}+ \begin{bmatrix} 1 & 2 \\ 6 & 4 \\ 7 & 3 \end{bmatrix} ^{\prime} \\ & =2 \begin{bmatrix} 1 & 4 & 5 \\ 2 & 1 & 6 \end{bmatrix} + \begin{bmatrix} 1 & 6 & 7 \\ 2 & 4 & 3 \end{bmatrix} \\ & = \begin{bmatrix} 2 & 8 & 10 \\ 4 & 2 & 12 \end{bmatrix} + \begin{bmatrix} 1 & 6 & 7 \\ 2 & 4 & 3 \end{bmatrix} \\ & = \begin{bmatrix} 2+1 & 8+6 & 10+7 \\ 4+2 & 2+4 & 12+3 \end{bmatrix} = \begin{bmatrix} 3 & 14 & 17 \\ 6 & 6 & 15 \end{bmatrix} \end{aligned} $

Hence, $\quad$ L.H.S. $=$ R.H.S.

$ (2 A+B)^{\prime}=2 A^{\prime}+B^{\prime} \text{ is verified. } $

(ii) To verify that: $(A-B)^{\prime}=A^{\prime}-B^{\prime}$

L.H.S. $(A-B)^{\prime}=[(\begin{matrix} 1 & 2 \\ 4 & 1 \\ 5 & 6\end{matrix} )-(\begin{matrix} 1 & 2 \\ 6 & 4 \\ 7 & 3\end{matrix} )]^{\prime}$

$= \begin{bmatrix} 1-1 & 2-2 \\ 4-6 & 1-4 \\ 5-7 & 6-3\end{bmatrix} ^{\prime}= \begin{bmatrix} 0 & 0 \\ -2 & -3 \\ -2 & 3\end{bmatrix} ^{\prime}= \begin{bmatrix} 0 & -2 & -2 \\ 0 & -3 & 3 \end{bmatrix} $

R.H.S. $A^{\prime}-B^{\prime}= \begin{bmatrix} 1 & 2 \\ 4 & 1 \\ 5 & 6\end{bmatrix} ^{\prime}- \begin{bmatrix} 1 & 2 \\ 6 & 4 \\ 7 & 3\end{bmatrix} ^{\prime}= \begin{bmatrix} 1 & 4 & 5 \\ 2 & 1 & 6\end{bmatrix} - \begin{bmatrix} 1 & 6 & 7 \\ 2 & 4 & 3 \end{bmatrix} $

$= \begin{bmatrix} 1-1 & 4-6 & 5-7 \\ 2-2 & 1-4 & 6-3\end{bmatrix} = \begin{bmatrix} 0 & -2 & -2 \\ 0 & -3 & 3 \end{bmatrix} $

Hence, $\quad$ L.H.S. $=$ R.H.S.

$(A-B)^{\prime}=A^{\prime}-B^{\prime}$ is verified.

29. Show that $A^{\prime} A$ and $AA^{\prime}$ are both symmetric matrices for any matrix A.

Show Answer

Solution

Let

$ P=A^{\prime} A $

$ \begin{matrix} \Rightarrow & P^{\prime}=(A^{\prime} A)^{\prime} \\ \Rightarrow & P^{\prime}=A^{\prime}(A^{\prime})^{\prime} \\ \Rightarrow & P^{\prime}=A^{\prime} A \\ \Rightarrow & P^{\prime}=P \end{matrix} $

Hence, $A^{\prime} A$ is a symmetric matrix.

Now, Let

$Q=AA^{\prime}$

$\Rightarrow \quad Q^{\prime}=(AA^{\prime})^{\prime}$

$\begin{matrix} \Rightarrow & Q^{\prime}=(A^{\prime})^{\prime} A^{\prime} \\ \Rightarrow & Q^{\prime}=AA^{\prime} \\ \Rightarrow & Q^{\prime}=Q\end{matrix} $

Hence, $AA^{\prime}$ is also a symmetric matrix.

30. Let $A$ and $B$ be square matrices of the order $3 \times 3$. Is $(A B)^{2}=$ $A^{2} B^{2}$ ? Give reasons.

Show Answer

Solution

Given that A and B are the matrices of the order $3 \times 3$.

$ \begin{aligned} (AB)^{2} & =AB \cdot AB \\ & =AA \cdot BB \\ & =A^{2} \cdot B^{2} \end{aligned} $

Hence, $\quad(A B)^{2}=A^{2} B^{2}$

31. Show that if $A$ and $B$ are square matrices such that $A B=B A$ then $(A+B)^{2}=A^{2}+2 A B+B^{2}$.

Show Answer

Solution

To prove that $(A+B)^{2}=A^{2}+2 A B+B^{2}$

$\begin{bmatrix} \text{ L.H.S. } \quad(A+B)^{2} & =(A+B) \cdot(A+B) & {\because \quad} & A^{2}=A \cdot A\end{bmatrix} $

So, $\quad$ L.H.S. $=$ R.H.S.

32. Let $A= \begin{bmatrix} 1 & 2 \\ -1 & 3\end{bmatrix} , B= \begin{bmatrix} 4 & 0 \\ 1 & 5\end{bmatrix} , C= \begin{bmatrix} 2 & 0 \\ 1 & -2 \end{bmatrix} $ and $a=4, b=-2$.

Show that:

(a) $A+(B+C)=(A+B)+C$

(b) $A(BC)=(AB) C$

(f) $(b A)^{T}=b A^{T}$

(c) $(a+b) B=a B+b B$

(g) $(AB)^{T}=B^{T} A^{T}$

(d) $a(C-A)=a C-a A$

(h) $(A-B) C=AC-BC$

(i) $(A-B)^{T}=A^{T}-B^{T}$

(e) $(A^{T})^{T}=A$

Show Answer

Solution

(a) To prove that: $A+(B+C)=(A+B)+C$

L.H.S. $A+(B+C)=(\begin{bmatrix} 1 & 2 \\ -1 & 3\end{bmatrix} )+[(\begin{bmatrix} 4 & 0 \\ 1 & 5\end{bmatrix} )+(\begin{bmatrix} 2 & 0 \\ 1 & -2\end{bmatrix} )]$

$= \begin{bmatrix} 1 & 2 \\ -1 & 3\end{bmatrix} + \begin{bmatrix} 4+2 & 0+0 \\ 1+1 & 5-2 \end{bmatrix} $

$= \begin{bmatrix} 1 & 2 \\ -1 & 3\end{bmatrix} + \begin{bmatrix} 6 & 0 \\ 2 & 3 \end{bmatrix} $

$= \begin{bmatrix} 1+6 & 2+0 \\ -1+2 & 3+3\end{bmatrix} = \begin{bmatrix} 7 & 2 \\ 1 & 6 \end{bmatrix} $

R.H.S. $(A+B)+C=[(\begin{bmatrix} 1 & 2 \\ -1 & 3\end{bmatrix} )+(\begin{bmatrix} 4 & 0 \\ 1 & 5\end{bmatrix} )]+(\begin{bmatrix} 2 & 0 \\ 1 & -2\end{bmatrix} )$

$= \begin{bmatrix} 1+4 & 2+0 \\ -1+1 & 3+5\end{bmatrix} + \begin{bmatrix} 2 & 0 \\ 1 & -2 \end{bmatrix} $

$= \begin{bmatrix} 5 & 2 \\ 0 & 8\end{bmatrix} + \begin{bmatrix} 2 & 0 \\ 1 & -2 \end{bmatrix} $

$= \begin{bmatrix} 5+2 & 2+0 \\ 0+1 & 8-2\end{bmatrix} = \begin{bmatrix} 7 & 2 \\ 1 & 6 \end{bmatrix} $

Hence, $\quad$ L.H.S. $=$ R.H.S.

$A+(B+C)=(A+B)+C$ Hence proved. (b) To prove that: $A(BC)=(AB) C$

$ \begin{aligned} \text{ L.H.S. } A(B C) & = \begin{bmatrix} 1 & 2 \\ -1 & 3 \end{bmatrix} [(\begin{matrix} 4 & 0 \\ 1 & 5 \end{matrix} )(\begin{matrix} 2 & 0 \\ 1 & -2 \end{matrix} )] \\ & = \begin{bmatrix} 1 & 2 \\ -1 & 3 \end{bmatrix} \begin{bmatrix} 8+0 & 0+0 \\ 2+5 & 0-10 \end{bmatrix} = \begin{bmatrix} 1 & 2 \\ -1 & 3 \end{bmatrix} \begin{bmatrix} 8 & 0 \\ 7 & -10 \end{bmatrix} \\ & = \begin{bmatrix} 8+14 & 0-20 \\ -8+21 & 0-30 \end{bmatrix} = \begin{bmatrix} 22 & -20 \\ 13 & -30 \end{bmatrix} \end{aligned} $

R.H.S. $(AB) C=[(\begin{bmatrix} 1 & 2 \\ -1 & 3\end{bmatrix} )(\begin{bmatrix} 4 & 0 \\ 1 & 5\end{bmatrix} )] \begin{bmatrix} 2 & 0 \\ 1 & -2 \end{bmatrix} $

$ = \begin{bmatrix} 4+2 & 0+10 \\ -4+3 & 0+15 \end{bmatrix} \begin{bmatrix} 2 & 0 \\ 1 & -2 \end{bmatrix} $

$ \begin{aligned} & = \begin{bmatrix} 6 & 10 \\ -1 & 15 \end{bmatrix} \begin{bmatrix} 2 & 0 \\ 1 & -2 \end{bmatrix} \\ & = \begin{bmatrix} 12+10 & 0-20 \\ -2+15 & 0-30 \end{bmatrix} = \begin{bmatrix} 22 & -20 \\ 13 & -30 \end{bmatrix} \end{aligned} $

Hence, $\quad$ L.H.S. $=$ R.H.S.

$ A(B C)=(A B) C \text{ Hence proved. } $

(c) To prove that: $(a+b) B=a B+b B$

Here, $a=4$ and $b=-2$

L.H.S. $\quad(a+b) B=(4-2) \begin{bmatrix} 4 & 0 \\ 1 & 5\end{bmatrix} =2 \begin{bmatrix} 4 & 0 \\ 1 & 5\end{bmatrix} = \begin{bmatrix} 8 & 0 \\ 2 & 10 \end{bmatrix} $

R.H.S. $a B+b B=4 \begin{bmatrix} 4 & 0 \\ 1 & 5\end{bmatrix} -2 \begin{bmatrix} 4 & 0 \\ 1 & 5\end{bmatrix} = \begin{bmatrix} 16 & 0 \\ 4 & 20\end{bmatrix} - \begin{bmatrix} 8 & 0 \\ 2 & 10 \end{bmatrix} $

$ = \begin{bmatrix} 16-8 & 0-0 \\ 4-2 & 20-10 \end{bmatrix} = \begin{bmatrix} 8 & 0 \\ 2 & 10 \end{bmatrix} $

Hence, L.H.S. = R.H.S.

$ (a+b) B=a B+b B \text{ Hence proved. } $

(d) To prove that: $a(C-A)=a C-a A$

L.H.S. $\quad a(C-A)=4[(\begin{bmatrix} 2 & 0 \\ 1 & -2\end{bmatrix} )-(\begin{bmatrix} 1 & 2 \\ -1 & 3\end{bmatrix} )]$

$ =4 \begin{bmatrix} 2-1 & 0-2 \\ 1+1 & -2-3 \end{bmatrix} =4 \begin{bmatrix} 1 & -2 \\ 2 & -5 \end{bmatrix} = \begin{bmatrix} 4 & -8 \\ 8 & -20 \end{bmatrix} $

R.H.S. $a C-a A=4 \begin{bmatrix} 2 & 0 \\ 1 & -2\end{bmatrix} -4 \begin{bmatrix} 1 & 2 \\ -1 & 3 \end{bmatrix} $

$ \begin{aligned} & = \begin{bmatrix} 8 & 0 \\ 4 & -8 \end{bmatrix} - \begin{bmatrix} 4 & 8 \\ -4 & 12 \end{bmatrix} \\ & = \begin{bmatrix} 8-4 & 0-8 \\ 4+4 & -8-12 \end{bmatrix} = \begin{bmatrix} 4 & -8 \\ 8 & -20 \end{bmatrix} \end{aligned} $

Hence, $\quad$ L.H.S. = R.H.S.

$ a(C-A)=a C-a A \text{ Hence proved. } $

(e) To prove that: $(A^{T})^{T}=A$

L.H.S.

$ \begin{aligned} A^{T} & = \begin{bmatrix} 1 & 2 \\ -1 & 3 \end{bmatrix} ^{T}= \begin{bmatrix} 1 & -1 \\ 2 & 3 \end{bmatrix} \\ (A^{T})^{T} & = \begin{bmatrix} 1 & -1 \\ 2 & 3 \end{bmatrix} ^{T}= \begin{bmatrix} 1 & 2 \\ -1 & 3 \end{bmatrix} =A \quad \text{ R.H.S. } \end{aligned} $

Hence, $\quad(A^{T})^{T}=A$

(f) To prove that: $(b A)^{T}=b A^{T}$ L.H.S. $(b A)^{T}=[-2(\begin{bmatrix} 1 & 2 \\ -1 & 3\end{bmatrix} )]^{T}= \begin{bmatrix} -2 & -4 \\ 2 & -6\end{bmatrix} ^{T}= \begin{bmatrix} -2 & 2 \\ -4 & -6 \end{bmatrix} $ R.H.S. $b A^{T}=-2 \begin{bmatrix} 1 & 2 \\ -1 & 3\end{bmatrix} ^{T}=-2 \begin{bmatrix} 1 & -1 \\ 2 & 3\end{bmatrix} = \begin{bmatrix} -2 & 2 \\ -4 & -6 \end{bmatrix} $

Hence, L.H.S. = R.H.S.

$ (b A)^{T}=b A^{T} \text{ Hence proved. } $

(g) To prove that: $(A B)^{T}=B^{T} A^{T}$

L.H.S. $(A B)^{T}=[(\begin{bmatrix} 1 & 2 \\ -1 & 3\end{bmatrix} )(\begin{bmatrix} 4 & 0 \\ 1 & 5\end{bmatrix} )]^{T}$

$ = \begin{bmatrix} 4+2 & 0+10 \\ -4+3 & 0+15 \end{bmatrix} ^{T}= \begin{bmatrix} 6 & 10 \\ -1 & 15 \end{bmatrix} ^{T}= \begin{bmatrix} 6 & -1 \\ 10 & 15 \end{bmatrix} $

R.H.S. $B^{T} A^{T}= \begin{bmatrix} 4 & 0 \\ 1 & 5\end{bmatrix} ^{T} \begin{bmatrix} 1 & 2 \\ -1 & 3 \end{bmatrix} ^{T}$

$ = \begin{bmatrix} 4 & 1 \\ 0 & 5 \end{bmatrix} \begin{bmatrix} 1 & -1 \\ 2 & 3 \end{bmatrix} = \begin{bmatrix} 4+2 & -4+3 \\ 0+10 & 0+15 \end{bmatrix} = \begin{bmatrix} 6 & -1 \\ 10 & 15 \end{bmatrix} $

Hence, $\quad$ L.H.S. = R.H.S.

$(A B)^{T}=B^{T} A^{T}$ Hence proved. (h) To prove that: $(A-B) C=AC-BC$

L.H.S. $\quad(A-B) C=[(\begin{bmatrix} 1 & 2 \\ -1 & 3\end{bmatrix} )-(\begin{bmatrix} 4 & 0 \\ 1 & 5\end{bmatrix} )] \begin{bmatrix} 2 & 0 \\ 1 & -2 \end{bmatrix} $

$= \begin{bmatrix} 1-4 & 2-0 \\ -1-1 & 3-5\end{bmatrix} \begin{bmatrix} 2 & 0 \\ 1 & -2 \end{bmatrix} $

$= \begin{bmatrix} -3 & 2 \\ -2 & -2\end{bmatrix} \begin{bmatrix} 2 & 0 \\ 1 & -2 \end{bmatrix} $

$= \begin{bmatrix} -6+2 & 0-4 \\ -4-2 & 0+4\end{bmatrix} = \begin{bmatrix} -4 & -4 \\ -6 & 4 \end{bmatrix} $

R.H.S. $AC-BC= \begin{bmatrix} 1 & 2 \\ -1 & 3\end{bmatrix} \begin{bmatrix} 2 & 0 \\ 1 & -2\end{bmatrix} - \begin{bmatrix} 4 & 0 \\ 1 & 5\end{bmatrix} \begin{bmatrix} 2 & 0 \\ 1 & -2 \end{bmatrix} $

$= \begin{bmatrix} 2+2 & 0-4 \\ -2+3 & 0-6\end{bmatrix} - \begin{bmatrix} 8+0 & 0+0 \\ 2+5 & 0-10 \end{bmatrix} $

$= \begin{bmatrix} 4 & -4 \\ 1 & -6\end{bmatrix} - \begin{bmatrix} 8 & 0 \\ 7 & -10 \end{bmatrix} $

$= \begin{bmatrix} 4-8 & -4-0 \\ 1-7 & -6+10\end{bmatrix} = \begin{bmatrix} -4 & -4 \\ -6 & 4 \end{bmatrix} $

Hence, $\quad$ L.H.S. $=$ R.H.S.

$ (A-B) C=A C-B C $

(i) To prove that: $(A-B)^{T}=A^{T}-B^{T}$

L.H.S. $\quad(A-B)^{T}=[(\begin{bmatrix} 1 & 2 \\ -1 & 3\end{bmatrix} )-(\begin{bmatrix} 4 & 0 \\ 1 & 5\end{bmatrix} )]^{T}$

$ = \begin{bmatrix} 1-4 & 2-0 \\ -1-1 & 3-5 \end{bmatrix} ^{T}= \begin{bmatrix} -3 & 2 \\ -2 & -2 \end{bmatrix} ^{T}= \begin{bmatrix} -3 & -2 \\ 2 & -2 \end{bmatrix} $

R.H.S. $\quad A^{T}-B^{T}= \begin{bmatrix} 1 & 2 \\ -1 & 3\end{bmatrix} ^{T}- \begin{bmatrix} 4 & 0 \\ 1 & 5\end{bmatrix} ^{T}= \begin{bmatrix} 1 & -1 \\ 2 & 3\end{bmatrix} - \begin{bmatrix} 4 & 1 \\ 0 & 5 \end{bmatrix} $

$ = \begin{bmatrix} 1-4 & -1-1 \\ 2-0 & 3-5 \end{bmatrix} = \begin{bmatrix} -3 & -2 \\ 2 & -2 \end{bmatrix} $

Hence, $\quad$ L.H.S. $=$ R.H.S.

$ (A-B)^{T}=A^{T}-B^{T} \text{ Hence proved. } $

33. If $A= \begin{bmatrix} \cos q & \sin q \\ -\sin q & \cos q \end{bmatrix} $, then show that

$ A^{2}= \begin{bmatrix} \cos 2 q & \sin 2 q \\ -\sin 2 q & \cos 2 q \end{bmatrix} $

Show Answer

Solution

Given that

$ \begin{aligned} A & = \begin{bmatrix} \cos q & \sin q \\ -\sin q & \cos q \end{bmatrix} \\ A=A \cdot A & = \begin{bmatrix} \cos q & \sin q \\ -\sin q & \cos q \end{bmatrix} \begin{bmatrix} \cos q & \sin q \\ -\sin q & \cos q \end{bmatrix} \\ & = \begin{bmatrix} \cos ^{2} q-\sin ^{2} q & \cos q \sin q+\sin q \cos q \\ \sin q \cos q-\cos q \sin q & -\sin ^{2} q+\cos ^{2} q \end{bmatrix} \\ & = \begin{bmatrix} \cos 2 q & \sin 2 q \\ -\sin 2 q & \cos 2 q \end{bmatrix} \begin{bmatrix} \because \cos ^{2} A-\sin ^{2} A=\cos 2 A \\ 2 \sin A \cos A=\sin 2 A \end{bmatrix} \end{aligned} $

Hence proved.

34. If $A= \begin{bmatrix} 0 & -x \\ x & 0\end{bmatrix} , B= \begin{bmatrix} 0 & 1 \\ 1 & 0 \end{bmatrix} $ and $x^{2}=-1$, then show that $(A+B)^{2}=A^{2}+B^{2}$.

Show Answer

Solution

Given that: $A= \begin{bmatrix} 0 & -x \\ x & 0\end{bmatrix} $ and $B= \begin{bmatrix} 0 & 1 \\ 1 & 0 \end{bmatrix} $

L.H.S. $\quad(A+B)^{2}=(A+B) \cdot(A+B)$

$ \begin{aligned} & =[(\begin{matrix} 0 & -x \\ x & 0 \end{matrix} )+(\begin{matrix} 0 & 1 \\ 1 & 0 \end{matrix} )] \cdot[(\begin{matrix} 0 & -x \\ x & 0 \end{matrix} )+(\begin{matrix} 0 & 1 \\ 1 & 0 \end{matrix} )] \\ & = \begin{bmatrix} 0+0 & -x+1 \\ x+1 & 0+0 \end{bmatrix} \cdot \begin{bmatrix} 0+0 & -x+1 \\ x+1 & 0+0 \end{bmatrix} \\ & = \begin{bmatrix} 0 & -x+1 \\ x+1 & 0 \end{bmatrix} \cdot \begin{bmatrix} 0 & -x+1 \\ x+1 & 0 \end{bmatrix} \\ & = \begin{bmatrix} 0+(-x+1)(x+1) & 0+0 \\ 0+0 & (x+1)(-x+1)+0 \end{bmatrix} \\ & = \begin{bmatrix} 1-x^{2} & 0 \\ 0 & 1-x^{2} \end{bmatrix} \end{aligned} $

Put $x^{2}=-1$

R.H.S.

$ = \begin{bmatrix} 1+1 & 0 \\ 0 & 1+1 \end{bmatrix} = \begin{bmatrix} 2 & 0 \\ 0 & 2 \end{bmatrix} $

(given)

$ \begin{aligned} A^{2}+B^{2} & =A \cdot A+B \cdot B \\ & = \begin{bmatrix} 0 & -x \\ x & 0 \end{bmatrix} \begin{bmatrix} 0 & -x \\ x & 0 \end{bmatrix} + \begin{bmatrix} 0 & 1 \\ 1 & 0 \end{bmatrix} \begin{bmatrix} 0 & 1 \\ 1 & 0 \end{bmatrix} \\ & = \begin{bmatrix} 0-x^{2} & 0+0 \\ 0+0 & -x^{2}+0 \end{bmatrix} + \begin{bmatrix} 0+1 & 0+0 \\ 0+0 & 1+0 \end{bmatrix} \\ & = \begin{bmatrix} -x^{2} & 0 \\ 0 & -x^{2} \end{bmatrix} + \begin{bmatrix} 1 & 0 \\ 0 & 1 \end{bmatrix} = \begin{bmatrix} -x^{2}+1 & 0+0 \\ 0+0 & -x^{2}+1 \end{bmatrix} \\ & = \begin{bmatrix} -x^{2}+1 & 0 \\ 0 & -x^{2}+1 \end{bmatrix} = \begin{bmatrix} 1+1 & 0 \\ 0 & 1+1 \end{bmatrix} [\because x^{2}=-1] \\ & = \begin{bmatrix} 2 & 0 \\ 0 & 2 \end{bmatrix} \end{aligned} $

Hence,

L.H.S. $=$ R.H.S.

$ (A+B)^{2}=A^{2}+B^{2} $

35. Verify that $A^{2}=I$ when $A= \begin{bmatrix} 0 & 1 & -1 \\ 4 & -3 & 4 \\ 3 & -3 & 4 \end{bmatrix} $.

Show Answer

Solution

Given that: $A= \begin{bmatrix} 0 & 1 & -1 \\ 4 & -3 & 4 \\ 3 & -3 & 4 \end{bmatrix} $

L.H.S. $\quad A^{2}=A \cdot A= \begin{bmatrix} 0 & 1 & -1 \\ 4 & -3 & 4 \\ 3 & -3 & 4\end{bmatrix} \begin{bmatrix} 0 & 1 & -1 \\ 4 & -3 & 4 \\ 3 & -3 & 4 \end{bmatrix} $

$ \begin{aligned} & = \begin{bmatrix} 0+4-3 & 0-3+3 & 0+4-4 \\ 0-12+12 & 4+9-12 & -4-12+16 \\ 0-12+12 & 3+9-12 & -3-12+16 \end{bmatrix} \\ & = \begin{bmatrix} 1 & 0 & 0 \\ 0 & 1 & 0 \\ 0 & 0 & 1 \end{bmatrix} =I \quad \text{ R.H.S. } \end{aligned} $

Hence, $A^{2}=I$ is verified.

36. Prove by Mathematical Induction that $(A^{\prime})^{n}=(A^{n})^{\prime}$, where $n \in N$ for any square matrix $A$.

Show Answer

Solution

To prove that $(A^{\prime})^{n}=(A^{n})^{\prime}$

Let $P(n):(A^{\prime})^{n}=(A^{n})^{\prime}$

Step 1: Put $n=1, P(1): A^{\prime}=A^{\prime} \quad$ which is true for $n=1$

Step 2: $\quad$ Put $n=K, P(K):(A^{\prime})^{K}=(A^{K})^{\prime} \quad$ Let it be true for $n=K$

Step 3: $\quad$ Put $n=K+1, P(K+1):(A^{\prime})^{K+1}=(A^{K+1})^{\prime}$

L.H.S.

$ \begin{aligned} (A^{\prime})^{K+1} & =(A^{\prime})^{K} \cdot(A^{\prime}) \\ & =(A^{K})^{\prime} \cdot(A^{\prime}) \\ & =(A^{K} \cdot A)^{\prime} \\ & =(A^{K+1})^{\prime} \quad \text{ R.H.S. } \end{aligned} $

(From step 2)

The given statement is true for $P(K+1)$ whenever it is true for $P(K)$, where $K \in N$.

37. Find inverse, by elementary row operations (if possible), of the following matrices.

(i) $ \begin{bmatrix} 1 & 3 \\ -5 & 7 \end{bmatrix} $

(ii) $ \begin{bmatrix} 1 & -3 \\ -2 & 6 \end{bmatrix} $

Show Answer

Solution

(i) Let

$ \begin{aligned} A & = \begin{bmatrix} 1 & 3 \\ -5 & 7 \end{bmatrix} \\ |A| & =1 \times 7-(-5) \times 3=7+15=22 \neq 0 \end{aligned} $

So, $A$ is invertible.

Let

$A=IA$

$ \begin{bmatrix} 1 & 3 \\ -5 & 7 \end{bmatrix} = \begin{bmatrix} 1 & 0 \\ 0 & 1 \end{bmatrix} A $

$R_2 \to R_2+5 R_1$

$\Rightarrow \quad \begin{bmatrix} 1 & 3 \\ 0 & 22\end{bmatrix} = \begin{bmatrix} 1 & 0 \\ 5 & 1 \end{bmatrix} A$

$R_2 \to \frac{1}{22} R_2$

$\Rightarrow \quad \begin{bmatrix} 1 & 3 \\ 0 & 1\end{bmatrix} = \begin{bmatrix} 1 & 0 \\ \frac{5}{22} & \frac{1}{22} \end{bmatrix} A$

$R_1 \to R_1-3 R_2$

$\Rightarrow \quad \begin{bmatrix} 1 & 0 \\ 0 & 1\end{bmatrix} = \begin{bmatrix} \frac{7}{22} & \frac{-3}{22} \\ \frac{5}{22} & \frac{1}{22} \end{bmatrix} A$

So

$ A^{-1}= \begin{bmatrix} \frac{7}{22} & \frac{-3}{22} \\ \frac{5}{22} & \frac{1}{22} \end{bmatrix} \Rightarrow \frac{1}{22} \begin{bmatrix} 7 & -3 \\ 5 & 1 \end{bmatrix} $

Hence, inverse of $ \begin{bmatrix} 1 & 3 \\ -5 & 7\end{bmatrix} $ is $\frac{1}{22} \begin{bmatrix} 7 & -3 \\ 5 & 1 \end{bmatrix} $

(ii) Let

$ \begin{aligned} A & = \begin{bmatrix} 1 & -3 \\ -2 & 6 \end{bmatrix} \\ |A| & =1 \times 6-(-3)(-2)=6-6=0 \\ |A| & =0 \text{ so } A \text{ is not invertible. } \end{aligned} $

Hence, inverse of $ \begin{bmatrix} 1 & -3 \\ -2 & 6 \end{bmatrix} $ is not possible.

38. If $ \begin{bmatrix} x y & 4 \\ z+6 & x+y\end{bmatrix} = \begin{bmatrix} 8 & w \\ 0 & 6 \end{bmatrix} $, then find the values of $x, y, z$ and $w$.

Show Answer

Solution

Given that: $ \begin{bmatrix} x y & 4 \\ z+6 & x+y\end{bmatrix} = \begin{bmatrix} 8 & w \\ 0 & 6 \end{bmatrix} $

Equating the corresponding elements,

$x y=8, w=4, z+6=0 \quad \Rightarrow \quad z=-6, x+y=6$

Now, solving $x+y=6$

and

$x y=8$

From eqn. (i), $\quad y=6-x$

Putting the value of $y$ in eqn. (ii) we get,

$ \begin{aligned} & x(6-x)=8 \Rightarrow 6 x-x^{2}=8 \\ & \Rightarrow x^{2}-6 x+8=0 \Rightarrow x^{2}-4 x-2 x+8=0 \\ & \Rightarrow x(x-4)-2(x-4)=0 \Rightarrow(x-4)(x-2)=0 \\ & \therefore \quad x=4,2 \end{aligned} $

39. If $A= \begin{bmatrix} 1 & 5 \\ 7 & 12\end{bmatrix} $ and $B= \begin{bmatrix} 9 & 1 \\ 7 & 8 \end{bmatrix} $, find a matrix $C$ such that $3 A+5 B+2 C$ is a null matrix.

Show Answer

Solution

Order of matrices A and B is $2 \times 2$.

$\therefore$ Order of matrix C must be $2 \times 2$.

$ \begin{aligned} & \text{ Let } C= \begin{bmatrix} a & b \\ c & d \end{bmatrix} \\ & \therefore \\ & \Rightarrow 3 \begin{bmatrix} 1 & 5 \\ 7 & 12 \end{bmatrix} +5 \begin{bmatrix} 9 & 1 \\ 7 & 8 \end{bmatrix} +2 \begin{bmatrix} a & b \\ c & d \end{bmatrix} = \begin{bmatrix} 0 & 0 \\ 0 & 0 \end{bmatrix} \\ & \Rightarrow \begin{bmatrix} 3 & 15 \\ 21 & 36 \end{bmatrix} + \begin{bmatrix} 45 & 5 \\ 35 & 40 \end{bmatrix} + \begin{bmatrix} 2 a & 2 b \\ 2 c & 2 d \end{bmatrix} = \begin{bmatrix} 0 & 0 \\ 0 & 0 \end{bmatrix} \\ & \Rightarrow \begin{bmatrix} 3+45+2 a & 15+5+2 b \\ 21+35+2 c & 36+40+2 d \end{bmatrix} = \begin{bmatrix} 0 & 0 \\ 0 & 0 \end{bmatrix} \\ & \Rightarrow \begin{bmatrix} 48+2 a & 20+2 b \\ 56+2 c & 76+2 d \end{bmatrix} = \begin{bmatrix} 0 & 0 \\ 0 & 0 \end{bmatrix} \end{aligned} $

Equating the corresponding elements, we get,

$ \begin{aligned} & 48+2 a=0 \Rightarrow 2 a=-48 \Rightarrow a=-24 \\ & 20+2 b=0 \Rightarrow 2 b=-20 \Rightarrow b=-10 \\ & 56+2 c=0 \Rightarrow 2 c=-56 \Rightarrow c=-28 \\ & 76+2 d=0 \Rightarrow 2 d=-76 \Rightarrow d=-38 \\ & C= \begin{bmatrix} -24 & -10 \\ -28 & -38 \end{bmatrix} \end{aligned} $

40. If $A= \begin{bmatrix} 3 & -5 \\ -4 & 2 \end{bmatrix} $, then find $A^{2}-5 A-14 I$. Hence, find $A^{3}$.

Show Answer

Solution

Given that: $\quad A= \begin{bmatrix} 3 & -5 \\ -4 & 2 \end{bmatrix} $

$ \begin{aligned} A^{2}=A \cdot A & = \begin{bmatrix} 3 & -5 \\ -4 & 2 \end{bmatrix} \begin{bmatrix} 3 & -5 \\ -4 & 2 \end{bmatrix} \\ & = \begin{bmatrix} 9+20 & -15-10 \\ -12-8 & 20+4 \end{bmatrix} = \begin{bmatrix} 29 & -25 \\ -20 & 24 \end{bmatrix} \\ \therefore \quad A^{2}-5 A-14 I & = \begin{bmatrix} 29 & -25 \\ -20 & 24 \end{bmatrix} -5 \begin{bmatrix} 3 & -5 \\ -4 & 2 \end{bmatrix} -14 \begin{bmatrix} 1 & 0 \\ 0 & 1 \end{bmatrix} \\ & = \begin{bmatrix} 29 & -25 \\ -20 & 24 \end{bmatrix} - \begin{bmatrix} 15 & -25 \\ -20 & 10 \end{bmatrix} - \begin{bmatrix} 14 & 0 \\ 0 & 14 \end{bmatrix} \end{aligned} $

$ \begin{aligned} & = \begin{bmatrix} 29 & -25 \\ -20 & 24 \end{bmatrix} - \begin{bmatrix} 29 & -25 \\ -20 & 24 \end{bmatrix} \\ & = \begin{bmatrix} 29-29 & -25+25 \\ -20+20 & 24-24 \end{bmatrix} = \begin{bmatrix} 0 & 0 \\ 0 & 0 \end{bmatrix} \end{aligned} $

Hence, $A^{2}-5 A-14 I=O$

Now, multiplying both sides by A, we get,

$ \begin{aligned} & A^{2} \cdot A-5 A \cdot A-14 IA=OA \\ & \Rightarrow \quad A^{3}-5 A^{2}-14 A=0 \\ & \Rightarrow \quad A^{3}=5 A^{2}+14 A \\ & \Rightarrow \quad A^{3}=5 \begin{bmatrix} 29 & -25 \\ -20 & 24 \end{bmatrix} +14 \begin{bmatrix} 3 & -5 \\ -4 & 2 \end{bmatrix} \\ & = \begin{bmatrix} 145 & -125 \\ -100 & 120 \end{bmatrix} + \begin{bmatrix} 42 & -70 \\ -56 & 28 \end{bmatrix} \\ & = \begin{bmatrix} 145+42 & -125-70 \\ -100-56 & 120+28 \end{bmatrix} = \begin{bmatrix} 187 & -195 \\ -156 & 148 \end{bmatrix} \end{aligned} $

Hence, $A^{3}= \begin{bmatrix} 187 & -195 \\ -156 & 148 \end{bmatrix}$

41. Find the values of $a, b, c$ and $d$ if

$ 3 \begin{bmatrix} a & b \\ c & d \end{bmatrix} = \begin{bmatrix} a & 6 \\ -1 & 2 d \end{bmatrix} + \begin{bmatrix} 4 & a+b \\ c+d & 3 \end{bmatrix} . $

Show Answer

Solution

Given that: $3 \begin{bmatrix} a & b \\ c & d\end{bmatrix} = \begin{bmatrix} a & 6 \\ -1 & 2 d\end{bmatrix} + \begin{bmatrix} 4 & a+b \\ c+d & 3 \end{bmatrix} $

$ \begin{bmatrix} 3 a & 3 b \\ 3 c & 3 d \end{bmatrix} = \begin{bmatrix} a+4 & 6+a+b \\ -1+c+d & 2 d+3 \end{bmatrix} $

Equating the corresponding elements, we get,

$ \begin{aligned} & 3 a=a+4 \quad \Rightarrow 3 a-a=4 \quad \Rightarrow 2 a=4 \quad \Rightarrow a=2 \\ & 3 b=6+a+b \quad \Rightarrow 3 b-b-a=6 \quad \Rightarrow 2 b-a=6 \quad \Rightarrow 2 b-2=6 \\ & \Rightarrow 2 b=8 \\ & \Rightarrow b=4 \\ & 3 c=-1+c+d \Rightarrow 3 c-c-d=-1 \Rightarrow 2 c-d=-1 \\ & \text{ and } 3 d=2 d+3 \Rightarrow 3 d-2 d=3 \quad \Rightarrow d=3 \\ & \text{ Now } 2 c-d=-1 \end{aligned} $

$ \begin{aligned} & \Rightarrow \quad 2 c-3=-1 \Rightarrow 2 c=3-1 \Rightarrow 2 c=2 \\ & \therefore \quad c=1 \\ & \therefore a=2, b=4, c=1 \text{ and } d=3 . \end{aligned} $

42. Find the matrix A such that

$ \begin{bmatrix} 2 & -1 \\ 1 & 0 \\ -3 & 4 \end{bmatrix} A= \begin{bmatrix} -1 & -8 & -10 \\ 1 & -2 & -5 \\ 9 & 22 & 15 \end{bmatrix} $

Show Answer

Solution

Order of matrix $ \begin{bmatrix} 2 & -1 \\ 1 & 0 \\ -3 & 4 \end{bmatrix} $ is $3 \times 2$ and the matrix

$ \begin{bmatrix} -1 & -8 & -10 \\ 1 & -2 & -5 \\ 9 & 22 & 15 \end{bmatrix} \text{ is } 3 \times 3 $

$\therefore$ Order of matrix A must be $2 \times 3$

Let $A= \begin{bmatrix} a & b & c \\ d & e & f \end{bmatrix} _{2 \times 3}$

So,$\begin{aligned} { \begin{bmatrix} 2 & -1 \\ 1 & 0 \\ -3 & 4\end{bmatrix} \begin{bmatrix} a & b & c \\ d & e & f\end{bmatrix} } & = \begin{bmatrix} -1 & -8 & -10 \\ 1 & -2 & -5 \\ 9 & 22 & 15\end{bmatrix} \\ { \begin{bmatrix} 2 a-d & 2 b-e & 2 c-f \\ a+0 & b+0 & c+0 \\ -3 a+4 d & -3 b+4 e & -3 c+4 f\end{bmatrix} } & = \begin{bmatrix} -1 & -8 & -10 \\ 1 & -2 & -5 \\ 9 & 22 & 15 \end{bmatrix} \end{aligned}$

Equating the corresponding elements, we get,

$ \begin{aligned} & 2 a-d=-1 \text{ and } a=1 \Rightarrow 2 \times 1-d=-1 \Rightarrow d=2+1 \Rightarrow d=3 \\ & 2 b-e=-8 \text{ and } b=-2 \Rightarrow 2(-2)-e=-8 \Rightarrow-4-e=-8 \\ & \Rightarrow e=4 \\ & 2 c-f=-10 \text{ and } c=-5 \Rightarrow 2(-5)-f=-10 \Rightarrow-10-f=-10 \\ & \Rightarrow f=0 \\ & a=1, b=-2, c=-5, d=3, e=4 \text{ and } f=0 \\ & A= \begin{bmatrix} 1 & -2 & -5 \\ 3 & 4 & 0 \end{bmatrix} \end{aligned} $

43. If $A= \begin{bmatrix} 1 & 2 \\ 4 & 1 \end{bmatrix} $, find $A^{2}+2 A+7 I$

Show Answer

Solution

Given that: $\quad A= \begin{bmatrix} 1 & 2 \\ 4 & 1 \end{bmatrix} $

$ \begin{aligned} A^{2}=A \cdot A & = \begin{bmatrix} 1 & 2 \\ 4 & 1 \end{bmatrix} \begin{bmatrix} 1 & 2 \\ 4 & 1 \end{bmatrix} = \begin{bmatrix} 1+8 & 2+2 \\ 4+4 & 8+1 \end{bmatrix} = \begin{bmatrix} 9 & 4 \\ 8 & 9 \end{bmatrix} \\ A^{2}+2 A+7 I & = \begin{bmatrix} 9 & 4 \\ 8 & 9 \end{bmatrix} +2 \begin{bmatrix} 1 & 2 \\ 4 & 1 \end{bmatrix} +7 \begin{bmatrix} 1 & 0 \\ 0 & 1 \end{bmatrix} \\ & = \begin{bmatrix} 9 & 4 \\ 8 & 9 \end{bmatrix} + \begin{bmatrix} 2 & 4 \\ 8 & 2 \end{bmatrix} + \begin{bmatrix} 7 & 0 \\ 0 & 7 \end{bmatrix} \\ & = \begin{bmatrix} 9+2+7 & 4+4+0 \\ 8+8+0 & 9+2+7 \end{bmatrix} = \begin{bmatrix} 18 & 8 \\ 16 & 18 \end{bmatrix} \end{aligned} $

Hence, $\quad A^{2}+2 A+7 I= \begin{bmatrix} 18 & 8 \\ 16 & 18 \end{bmatrix} $.

44. If $A= \begin{bmatrix} \cos \alpha & \sin \alpha \\ -\sin \alpha & \cos \alpha \end{bmatrix} $, and $A^{-1}=A^{\prime}$, find value of $\alpha$.

Show Answer

Solution

Here,

$ A= \begin{bmatrix} \cos \alpha & \sin \alpha \\ -\sin \alpha & \cos \alpha \end{bmatrix} $

Given that: $\quad A^{-1}=A^{\prime}$

Pre-multiplying both sides by $A$

$ AA^{-1}=AA^{\prime} $

$\Rightarrow \quad I=AA^{\prime}$

$\Rightarrow \begin{bmatrix} 1 & 0 \\ 0 & 1\end{bmatrix} = \begin{bmatrix} \cos \alpha & \sin \alpha \\ -\sin \alpha & \cos \alpha\end{bmatrix} \begin{bmatrix} \cos \alpha & -\sin \alpha \\ \sin \alpha & \cos \alpha \end{bmatrix} $

$\Rightarrow \begin{bmatrix} 1 & 0 \\ 0 & 1\end{bmatrix} = \begin{bmatrix} \cos ^{2} \alpha+\sin ^{2} \alpha & -\sin \alpha \cos \alpha+\sin \alpha \cos \alpha \\ -\sin \alpha \cos \alpha+\cos \alpha \sin \alpha & \sin ^{2} \alpha+\cos ^{2} \alpha \end{bmatrix} $

$\Rightarrow \begin{bmatrix} 1 & 0 \\ 0 & 1\end{bmatrix} = \begin{bmatrix} 1 & 0 \\ 0 & 1 \end{bmatrix} $

Hence, it is true for all values of $\alpha$.

45. If the matrix $ \begin{bmatrix} 0 & a & 3 \\ 2 & b & -1 \\ c & 1 & 0 \end{bmatrix} $ is a skew symmetric matrix, find the values of $a, b$ and $c$.

Show Answer

Solution

Let $\quad A= \begin{bmatrix} 0 & a & 3 \\ 2 & b & -1 \\ c & 1 & 0\end{bmatrix} A^{\prime}= \begin{bmatrix} 0 & 2 & c \\ a & b & 1 \\ 3 & -1 & 0 \end{bmatrix} $

For skew symmetric matrix, $A^{\prime}=-A$.

$ \begin{aligned} & { \begin{bmatrix} 0 & 2 & c \\ a & b & 1 \\ 3 & -1 & 0 \end{bmatrix} =- \begin{bmatrix} 0 & a & 3 \\ 2 & b & -1 \\ c & 1 & 0 \end{bmatrix} } \\ & \Rightarrow \quad \begin{bmatrix} 0 & 2 & c \\ a & b & 1 \\ 3 & -1 & 0 \end{bmatrix} = \begin{bmatrix} 0 & -a & -3 \\ -2 & -b & 1 \\ -c & -1 & 0 \end{bmatrix} \end{aligned} $

Equating the corresponding elements, we get

$a=-2, b=-b \Rightarrow 2 b=0 \Rightarrow b=0$ and $c=-3$

Hence, $a=-2, b=0$ and $c=-3$.

46. If $P(x)= \begin{bmatrix} \cos x & \sin x \\ -\sin x & \cos x \end{bmatrix} $, then show that

$ P(x) \cdot P(y)=P(x+y)=P(y) \cdot P(x) $

Show Answer

Solution

Given that:

$ \begin{aligned} P(x) & = \begin{bmatrix} \cos x & \sin x \\ -\sin x & \cos x \end{bmatrix} \\ P(y) & = \begin{bmatrix} \cos y & \sin y \\ -\sin y & \cos y \end{bmatrix} \\ P(x) \cdot P(y) & = \begin{bmatrix} \cos x & \sin x \\ -\sin x & \cos x \end{bmatrix} \begin{bmatrix} \cos y & \sin y \\ -\sin y & \cos y \end{bmatrix} \\ & = \begin{bmatrix} \cos x \cos y-\sin x \sin y & \cos x \sin y+\sin x \cos y \\ -\sin x \cos y-\cos x \sin y & -\sin x \sin y+\cos x \cos y \end{bmatrix} \\ & = \begin{bmatrix} \cos (x+y) & \sin (x+y) \\ -\sin (x+y) & \cos (x+y) \end{bmatrix} \\ & =P(x+y) \end{aligned} $

Now

$ \begin{aligned} P(y) \cdot P(x) & = \begin{bmatrix} \cos y & \sin y \\ -\sin y & \cos y \end{bmatrix} \begin{bmatrix} \cos x & \sin x \\ -\sin x & \cos x \end{bmatrix} \\ & = \begin{bmatrix} \cos x \cos y-\sin x \sin y & \sin x \cos y+\cos x \sin y \\ -\cos x \sin y-\cos y \sin x & -\sin x \sin y+\cos x \cos y \end{bmatrix} \end{aligned} $

$ \begin{aligned} & = \begin{bmatrix} \cos (x+y) & \sin (x+y) \\ -\sin (x+y) & \cos (x+y) \end{bmatrix} \\ & =P(x+y) \end{aligned} $

Hence, $P(x) \cdot P(y)=P(x+y)=P(y) \cdot P(x)$.

47. If $A$ is a square matrix such that $A^{2}=A$, show that $(I+A)^{3}=7 A+I$.

Show Answer

Solution

To show that: $(I+A)^{3}=7 A+I$

L.H.S. $\quad(I+A)^{3}=I^{3}+A^{3}+3 I^{2} A+3 I A^{2}$

$ \begin{aligned} & \Rightarrow I+A^{2} \cdot A+3 IA+3 IA^{2} \\ & \Rightarrow I+A \cdot A+3 IA+3 IA \\ & {[\because A^{2}=A]} \\ & \Rightarrow I+A^{2}+3 IA+3 IA \\ & \Rightarrow I+A+3 IA+3 IA \quad[\because A^{2}=A] \\ & \Rightarrow I+A+3 A+3 A \Rightarrow 7 A+I \quad \text{ R.H.S. } \end{aligned} $

L.H.S. $=$ R.H.S. Hence, Proved.

48. If $A$ and $B$ are square matrices of the same order and $B$ is a skew symmetric matrix, show that $A^{\prime} BA$ is a skew symmetric.

Show Answer

Solution

Given that $B$ is a skew symmetric matrix $\therefore B^{\prime}=-B$

Let

$ \begin{aligned} P & =A^{\prime} BA \\ P^{\prime} & =(A^{\prime} BA)^{\prime} \\ & =A^{\prime} B^{\prime}(A^{\prime})^{\prime} \\ & =A^{\prime}(-B) A \\ & =-A^{\prime} BA=-P \\ P^{\prime} & =-P \end{aligned} $

$ \Rightarrow $

So

Hence, $A^{\prime} BA$ is a skew symmetric matrix.

Long Answer Type Questions

49. If $A B=B A$ for any two square matrices, prove by mathematical induction that $(A B)^{n}=A^{n} B^{n}$.

Show Answer

Solution

Let $P(n):(AB)^{n}=A^{n} B^{n}$

Step 1:

Put $n=1, P(1): AB=AB$

Step 2:

Put $n=k, P(k):(AB)^{k}=A^{k} B^{k} \quad$ (Let it be true for any $k \in N$ )

Step 3:

Put $n=k+1, P(k+1):(AB)^{k+1}=A^{k+1} B^{k+1}$

L.H.S.

$ \begin{aligned} (AB)^{k+1} & =(AB)^{k} \cdot AB \\ & =A^{k} B^{k} \cdot AB \\ & =A^{k+1} B^{k+1} \quad \text{ R.H.S. } \end{aligned} $

$ =A^{k} B^{k} \cdot AB \quad[\text{ from Step 2] } $

L.H.S. $=$ R.H.S.

Hence, if $P(n)$ is true for $P(k)$ then it is true for $P(k+1)$.

50. Find $x, y, z$ if $A= \begin{bmatrix} 0 & 2 y & z \\ x & y & -z \\ x & -y & z \end{bmatrix} $ satisfies $A^{\prime}=A^{-1}$.

Show Answer

Solution

Given that: $\quad A= \begin{bmatrix} 0 & 2 y & z \\ x & y & -z \\ x & -y & z \end{bmatrix} $ and $A^{\prime}=A^{-1}$

Pre-multiplying both sides by $A$ we get,

$ \begin{gathered} AA^{\prime}=AA^{-1} \\ \Rightarrow \\ \Rightarrow \begin{bmatrix} 0 & 2 y & z \\ x & y & -z \\ x & -y & z \end{bmatrix} \begin{bmatrix} 0 & x & x \\ 2 y & y & -y \\ z & -z & z \end{bmatrix} = \begin{bmatrix} 1 & 0 & 0 \\ 0 & 1 & 0 \\ 0 & 0 & 1 \end{bmatrix} \\ \Rightarrow \begin{bmatrix} 0+4 y^{2}+z^{2} & 0+2 y^{2}-z^{2} & 0-2 y^{2}+z^{2} \\ 0+2 y^{2}-z^{2} & x^{2}+y^{2}+z^{2} & x^{2}-y^{2}-z^{2} \\ 0-2 y^{2}+z^{2} & x^{2}-y^{2}-z^{2} & x^{2}+y^{2}+z^{2} \end{bmatrix} = \begin{bmatrix} 1 & 0 & 0 \\ 0 & 1 & 0 \\ 0 & 0 & 1 \end{bmatrix} \end{gathered} $

Equating the corresponding elements, we get,

$$ \begin{align*} & 4 y^{2}+z^{2}=1 \tag{i}\\ & 2 y^{2}-z^{2}=0 \tag{ii} \end{align*} $$

Adding ( $i$ ) and (ii) we get, $6 y^{2}=1 \Rightarrow y^{2}=\frac{1}{6} \Rightarrow y= \pm \frac{1}{\sqrt{6}}$

From eqn. (i), we get,

$$ \begin{align*} 4 y^{2}+z^{2} & =1 \\ \Rightarrow 4(\frac{1}{\sqrt{6}})^{2}+z^{2} & =1 \Rightarrow \frac{2}{3}+z^{2}=1 \Rightarrow z^{2}=1-\frac{2}{3}=\frac{1}{3} \\ \therefore \quad z & = \pm \frac{1}{\sqrt{3}} \\ x^{2}+y^{2}+z^{2} & =1 \tag{iii}\\ x^{2}-y^{2}-z^{2} & =0 \tag{iv} \end{align*} $$

Adding (iii) and (iv) we get,

$ 2 x^{2}=1 \Rightarrow x^{2}=\frac{1}{2} \Rightarrow x= \pm \frac{1}{\sqrt{2}} $

Hence, $x= \pm \frac{1}{\sqrt{ }}, y= \pm \frac{1}{\sqrt{ }} \underset{6}{and} z= \pm \frac{1}{\sqrt{ }}$.

51. If possible, using elementary row transformation, find the inverse of the following matrices.

(i) $ \begin{bmatrix} 2 & -1 & 3 \\ -5 & 3 & 1 \\ -3 & 2 & 3 \end{bmatrix} $

(ii) $ \begin{bmatrix} 2 & 3 & -3 \\ -1 & -2 & 2 \\ 1 & 1 & 1 \end{bmatrix} $

(iii) $ \begin{bmatrix} 2 & 0 & -1 \\ 5 & 1 & 0 \\ 0 & 1 & 3 \end{bmatrix} $

Show Answer

Solution

(i) Here, $A= \begin{bmatrix} 2 & -1 & 3 \\ -5 & 3 & 1 \\ -3 & 2 & 3 \end{bmatrix} $ for elementary row transformation we put

$A=IA$

$ \begin{bmatrix} 2 & -1 & 3 \\ -5 & 3 & 1 \\ -3 & 2 & 3\end{bmatrix} = \begin{bmatrix} 1 & 0 & 0 \\ 0 & 1 & 0 \\ 0 & 0 & 1 \end{bmatrix} A$

$R_2 \to R_2+R_1$

$ \begin{bmatrix} 2 & -1 & 3 \\ -3 & 2 & 4 \\ -3 & 2 & 3\end{bmatrix} = \begin{bmatrix} 1 & 0 & 0 \\ 1 & 1 & 0 \\ 0 & 0 & 1 \end{bmatrix} A$

$R_3 \to R_3-R_2$

$ \begin{bmatrix} 2 & -1 & 3 \\ -3 & 2 & 4 \\ 0 & 0 & -1\end{bmatrix} = \begin{bmatrix} 1 & 0 & 0 \\ 1 & 1 & 0 \\ -1 & -1 & 1 \end{bmatrix} A$

$R_1 \to R_1+R_2$

$ \begin{bmatrix} -1 & 1 & 7 \\ -3 & 2 & 4 \\ 0 & 0 & -1\end{bmatrix} = \begin{bmatrix} 2 & 1 & 0 \\ 1 & 1 & 0 \\ -1 & -1 & 1 \end{bmatrix} A$

$R_2 \to R_2-3 R_1$

$ \begin{bmatrix} -1 & 1 & 7 \\ 0 & -1 & -17 \\ 0 & 0 & -1\end{bmatrix} = \begin{bmatrix} 2 & 1 & 0 \\ -5 & -2 & 0 \\ -1 & -1 & 1 \end{bmatrix} A$

$R_1 \to R_1+R_2$ and $R_3 \to-1 . R_3$

$ \begin{bmatrix} -1 & 0 & -10 \\ 0 & -1 & -17 \\ 0 & 0 & 1\end{bmatrix} = \begin{bmatrix} -3 & -1 & 0 \\ -5 & -2 & 0 \\ 1 & 1 & -1 \end{bmatrix} A$

$ \begin{aligned} & R_1 \to R_1+10 R_3 \text{ and } R_2 \to R_2+17 R_3 \\ & { \begin{bmatrix} -1 & 0 & 0 \\ 0 & -1 & 0 \\ 0 & 0 & 1 \end{bmatrix} = \begin{bmatrix} 7 & 9 & -10 \\ 12 & 15 & -17 \\ 1 & 1 & -1 \end{bmatrix} A} \\ & R_1 \to-1 \cdot R_1 \text{ and } R_2 \to-1 . R_2 \\ & { \begin{bmatrix} 1 & 0 & 0 \\ 0 & 1 & 0 \\ 0 & 0 & 1 \end{bmatrix} = \begin{bmatrix} -7 & -9 & 10 \\ -12 & -15 & 17 \\ 1 & 1 & -1 \end{bmatrix} A} \\ & A^{-1}= \begin{bmatrix} -7 & -9 & 10 \\ -12 & -15 & 17 \\ 1 & 1 & -1 \end{bmatrix} \\ & A= \begin{bmatrix} 2 & 3 & -3 \\ -1 & -2 & 2 \\ 1 & 1 & -1 \end{bmatrix} \\ & \text{ Put } \quad A=IA \\ & { \begin{bmatrix} 2 & 3 & -3 \\ -1 & -2 & 2 \\ 1 & 1 & -1 \end{bmatrix} = \begin{bmatrix} 1 & 0 & 0 \\ 0 & 1 & 0 \\ 0 & 0 & 1 \end{bmatrix} A} \\ & R_1 \to R_1-2 R_3 \text{ and } R_2 \to R_2+R_1 \\ & { \begin{bmatrix} 0 & 1 & -1 \\ 0 & -1 & 1 \\ 1 & 1 & -1 \end{bmatrix} = \begin{bmatrix} 1 & 0 & -2 \\ 0 & 1 & 1 \\ 0 & 0 & 1 \end{bmatrix} A} \\ & R_1 \to R_1+R_2 \\ & { \begin{bmatrix} 0 & 0 & 0 \\ 0 & -1 & 1 \\ 1 & 1 & -1 \end{bmatrix} = \begin{bmatrix} 1 & 1 & -1 \\ 0 & 1 & 1 \\ 0 & 0 & 1 \end{bmatrix} A} \end{aligned} $

First row on L.H.S. contains all zeros, so the inverse of the given matrix A does not exist.

Hence, matrix A has no inverse.

(iii) Here,

$ A= \begin{bmatrix} 2 & 0 & -1 \\ 5 & 1 & 0 \\ 0 & 1 & 3 \end{bmatrix} $

$ \begin{aligned} & \text{ Put } \quad A=I A \\ & { \begin{bmatrix} 2 & 0 & -1 \\ 5 & 1 & 0 \\ 0 & 1 & 3 \end{bmatrix} = \begin{bmatrix} 1 & 0 & 0 \\ 0 & 1 & 0 \\ 0 & 0 & 1 \end{bmatrix} A} \\ & R_1 \to 3 R_1-R_2 \\ & { \begin{bmatrix} 1 & -1 & -3 \\ 5 & 1 & 0 \\ 0 & 1 & 3 \end{bmatrix} = \begin{bmatrix} 3 & -1 & 0 \\ 0 & 1 & 0 \\ 0 & 0 & 1 \end{bmatrix} A} \\ & R_2 \to R_2-5 R_1 \\ & { \begin{bmatrix} 1 & -1 & -3 \\ 0 & 6 & 15 \\ 0 & 1 & 3 \end{bmatrix} = \begin{bmatrix} 3 & -1 & 0 \\ -15 & 6 & 0 \\ 0 & 0 & 1 \end{bmatrix} A} \\ & R_2 \to R_2-5 R_3 \\ & { \begin{bmatrix} 1 & -1 & -3 \\ 0 & 1 & 0 \\ 0 & 1 & 3 \end{bmatrix} = \begin{bmatrix} 3 & -1 & 0 \\ -15 & 6 & -5 \\ 0 & 0 & 1 \end{bmatrix} A} \\ & R_3 \to R_3-R_2 \\ & { \begin{bmatrix} 1 & -1 & -3 \\ 0 & 1 & 0 \\ 0 & 0 & 3 \end{bmatrix} = \begin{bmatrix} 3 & -1 & 0 \\ -15 & 6 & -5 \\ 15 & -6 & 6 \end{bmatrix} A} \\ & R_1 \to R_1+R_2 \\ & { \begin{bmatrix} 1 & 0 & -3 \\ 0 & 1 & 0 \\ 0 & 0 & 3 \end{bmatrix} = \begin{bmatrix} -12 & 5 & -5 \\ -15 & 6 & -5 \\ 15 & -6 & 6 \end{bmatrix} A} \\ & R_3 \to \frac{1}{3} R_3 \\ & { \begin{bmatrix} 1 & 0 & -3 \\ 0 & 1 & 0 \\ 0 & 0 & 1 \end{bmatrix} = \begin{bmatrix} -12 & 5 & -5 \\ -15 & 6 & -5 \\ 5 & -2 & 2 \end{bmatrix} A} \\ & R_1 \to R_1+3 R_3 \\ & { \begin{bmatrix} 1 & 0 & 0 \\ 0 & 1 & 0 \\ 0 & 0 & 1 \end{bmatrix} = \begin{bmatrix} 3 & -1 & 1 \\ -15 & 6 & -5 \\ 5 & -2 & 2 \end{bmatrix} A} \end{aligned} $

Hence,

$ A^{-1}= \begin{bmatrix} 3 & -1 & 1 \\ -15 & 6 & -5 \\ 5 & -2 & 2 \end{bmatrix} $

52. Express the matrix $ \begin{bmatrix} 2 & 3 & 1 \\ 1 & -1 & 2 \\ 4 & 1 & 2 \end{bmatrix} $ as the sum of symmetric and a skew symmetric matrix.

Show Answer

Solution

We know that any square matrix can be expressed as the sum of symmetric and skew symmetric matrix i.e. $A=\frac{1}{2}[A+A^{\prime}]+\frac{1}{2}[A-A^{\prime}]$.

Let $P=\frac{1}{2}[A+A^{\prime}]$ and $Q=\frac{1}{2}[A-A^{\prime}]$

So

$ \begin{aligned} P & =\frac{1}{2}[(\begin{matrix} 2 & 3 & 1 \\ 1 & -1 & 2 \\ 4 & 1 & 2 \end{matrix} )+(\begin{matrix} 2 & 1 & 4 \\ 3 & -1 & 1 \\ 1 & 2 & 2 \end{matrix} )] \\ & =\frac{1}{2} \begin{bmatrix} 2+2 & 3+1 & 1+4 \\ 1+3 & -1-1 & 2+1 \\ 4+1 & 1+2 & 2+2 \end{bmatrix} =\frac{1}{2} \begin{bmatrix} 4 & 4 & 5 \\ 4 & -2 & 3 \\ 5 & 3 & 4 \end{bmatrix} \\ & = \begin{bmatrix} 2 & 2 & 5 / 2 \\ 2 & -1 & 3 / 2 \\ 5 / 2 & 3 / 2 & 2 \end{bmatrix} \\ P^{\prime} & = \begin{bmatrix} 2 & 2 & 5 / 2 \\ 2 & -1 & 3 / 2 \\ 5 / 2 & 3 / 2 & 2 \end{bmatrix} =P \end{aligned} $

As $P^{\prime}=P \quad \therefore P$ is a symmetric matrix.

Now $Q=\frac{1}{2}[A-A^{\prime}]$

$ \begin{aligned} & =\frac{1}{2}[(\begin{matrix} 2 & 3 & 1 \\ 1 & -1 & 2 \\ 4 & 1 & 2 \end{matrix} )-(\begin{matrix} 2 & 1 & 4 \\ 3 & -1 & 1 \\ 1 & 2 & 2 \end{matrix} )] \\ & =\frac{1}{2}[(\begin{matrix} 2-2 & 3-1 & 1-4 \\ 1-3 & -1+1 & 2-1 \\ 4-1 & 1-2 & 2-2 \end{matrix} )]=\frac{1}{2}[(\begin{matrix} 0 & 2 & -3 \\ -2 & 0 & 1 \\ 3 & -1 & 0 \end{matrix} )] \end{aligned} $

$ = \begin{bmatrix} 0 & 1 & -3 / 2 \\ -1 & 0 & 1 / 2 \\ 3 / 2 & -1 / 2 & 0 \end{bmatrix} =- \begin{bmatrix} 0 & -1 & 3 / 2 \\ 1 & 0 & -1 / 2 \\ -3 / 2 & 1 / 2 & 0 \end{bmatrix} =-Q . $

As $Q=-Q \quad \therefore Q$ is a skew symmetric matrix.

So

$ \begin{aligned} A & =P+Q \\ A & = \begin{bmatrix} 2 & 2 & 5 / 2 \\ 2 & -1 & 3 / 2 \\ 5 / 2 & 3 / 2 & 2 \end{bmatrix} + \begin{bmatrix} 0 & 1 & -3 / 2 \\ -1 & 0 & 1 / 2 \\ 3 / 2 & -1 / 2 & 0 \end{bmatrix} \\ & = \begin{bmatrix} 2+0 & 2+1 & \frac{5}{2}-\frac{3}{2} \\ 2-1 & -1+0 & \frac{3}{2}+\frac{1}{2} \\ \frac{5}{2}+\frac{3}{2} & \frac{3}{2}-\frac{1}{2} & 2+0 \end{bmatrix} = \begin{bmatrix} 2 & 3 & 1 \\ 1 & -1 & 2 \\ 4 & 1 & 2 \end{bmatrix} =A \end{aligned} $

Hence, the required relation is

$ A= \begin{bmatrix} 2 & 2 & 5 / 2 \\ 2 & -1 & 3 / 2 \\ 5 / 2 & 3 / 2 & 2 \end{bmatrix} + \begin{bmatrix} 0 & 1 & -3 / 2 \\ -1 & 0 & 1 / 2 \\ 3 / 2 & -1 / 2 & 0 \end{bmatrix} $

Objective Type Questions

53. The matrix $P= \begin{bmatrix} 0 & 0 & 4 \\ 0 & 4 & 0 \\ 4 & 0 & 0 \end{bmatrix} $ is a

(a) square matrix

(b) diagonal matrix

(c) unit matrix

(d) None

Show Answer

Solution

Given that $A= \begin{bmatrix} 0 & 0 & 4 \\ 0 & 4 & 0 \\ 4 & 0 & 0 \end{bmatrix} $

Here number of columns and the number of rows are equal i.e., 3. So, $A$ is a square matrix.

Hence, the correct option is $(a)$.

  • Option (b) Diagonal matrix: A diagonal matrix is a square matrix in which all the elements outside the main diagonal are zero. In the given matrix ( P ), the elements outside the main diagonal are not zero (e.g., ( P_{1,3} = 4 ), ( P_{3,1} = 4 )), so it is not a diagonal matrix.

  • Option (c) Unit matrix: A unit matrix, also known as an identity matrix, is a square matrix in which all the elements on the main diagonal are 1 and all other elements are 0. In the given matrix ( P ), the elements on the main diagonal are not all 1 (e.g., ( P_{2,2} = 4 )), so it is not a unit matrix.

  • Option (d) None: This option is incorrect because the matrix ( P ) is indeed a square matrix, as established in the given answer.

54. Total number of possible matrices of order $3 \times 3$ with each entry 2 or 0 is

(a) 9

(b) 27

(c) 81

(d) 512

Show Answer

Solution

Total number of possible matrices of order $3 \times 3$ with each entry 0 or $2=2^{3 \times 3}=2^{9}=512$.

Hence, the correct option is $(d)$.

  • Option (a) 9 is incorrect because it does not account for the exponential growth of combinations. The number of possible matrices is not simply the number of entries (9), but rather the number of combinations of 0s and 2s for each entry.

  • Option (b) 27 is incorrect because it suggests that each entry has only 3 possible values (which would be 3^3 for a 3x3 matrix), but in this problem, each entry has only 2 possible values (0 or 2).

  • Option (c) 81 is incorrect because it implies that each entry has 4 possible values (which would be 4^2 for a 3x3 matrix), but in this problem, each entry has only 2 possible values (0 or 2).

55. If $ \begin{bmatrix} 2 x+y & 4 x \\ 5 x-7 & 4 x\end{bmatrix} = \begin{bmatrix} 7 & 7 y-13 \\ y & x+6 \end{bmatrix} $, then the value of $x$ and $y$ is

(a) $x=3, y=1$

(b) $x=2, y=3$

(c) $x=2, y=4$

(d) $x=3, y=3$

Show Answer

Solution

Given that: $ \begin{bmatrix} 2 x+y & 4 x \\ 5 x-7 & 4 x\end{bmatrix} = \begin{bmatrix} 7 & 7 y-13 \\ y & x+6 \end{bmatrix} $

Equating the corresponding elements, we get,

$\qquad 2 x+y=7 \qquad ……(1)$

$ \begin{aligned} & \text{ and } \quad 4 x=x+6 \qquad ……(2) \\ & \text{ from eqn. (ii) } \quad 4 x-x=6 \\ & 3 x=6 \\ & \therefore \quad x=2 \\ & \text{ from eqn. (i) } 2 \times 2+y=7 \\ & 4+y=7 \quad \therefore y=7-4=3 \end{aligned} $

Hence, the correct option is (b).

  • Option (a) $x=3, y=1$:

    • Substituting $x=3$ and $y=1$ into the first equation $2x + y = 7$:
      • $2(3) + 1 = 6 + 1 = 7$ (This is correct)
    • Substituting $x=3$ into the second equation $4x = x + 6$:
      • $4(3) = 3 + 6$
      • $12 \neq 9$ (This is incorrect)
    • Therefore, $x=3, y=1$ does not satisfy both equations.
  • Option (c) $x=2, y=4$:

    • Substituting $x=2$ and $y=4$ into the first equation $2x + y = 7$:
      • $2(2) + 4 = 4 + 4 = 8$ (This is incorrect)
    • Substituting $x=2$ into the second equation $4x = x + 6$:
      • $4(2) = 2 + 6$
      • $8 = 8$ (This is correct)
    • Therefore, $x=2, y=4$ does not satisfy both equations.
  • Option (d) $x=3, y=3$:

    • Substituting $x=3$ and $y=3$ into the first equation $2x + y = 7$:
      • $2(3) + 3 = 6 + 3 = 9$ (This is incorrect)
    • Substituting $x=3$ into the second equation $4x = x + 6$:
      • $4(3) = 3 + 6$
      • $12 \neq 9$ (This is incorrect)
    • Therefore, $x=3, y=3$ does not satisfy both equations.

56. If

$ \begin{aligned} & A=\frac{1}{\pi} \begin{bmatrix} \sin ^{-1}(x \pi) & \tan ^{-1}(\frac{x}{\pi}) \\ \sin ^{-1}(\frac{x}{\pi}) & \cot ^{-1}(\pi x) \end{bmatrix} \\ & B=\frac{1}{\pi} \begin{bmatrix} -\cos ^{-1}(x \pi) & \tan ^{-1}(\frac{x}{\pi}) \\ \sin ^{-1}(\frac{x}{\pi}) & -\tan ^{-1}(\pi x) \end{bmatrix} \end{aligned} $

then $A-B$ is equal to

(a) I

(b) $O$

(c) $2 I$

(d) $\frac{1}{2} I$

Show Answer

Solution

Given that: $A=\frac{1}{\pi} \begin{bmatrix} \sin ^{-1}(x \pi) & \tan ^{-1}(\frac{x}{\pi}) \\ \sin ^{-1}(\frac{x}{\pi}) & \cot ^{-1}(\pi x) \end{bmatrix} $

and

$ B=\frac{1}{\pi} \begin{bmatrix} -\cos ^{-1}(x \pi) & \tan ^{-1}(\frac{x}{\pi}) \\ \sin ^{-1}(\frac{x}{\pi}) & -\tan ^{-1}(\pi x) \end{bmatrix} $

$ \begin{aligned} & A-B=\frac{1}{\pi} \begin{bmatrix} \sin ^{-1}(x \pi) & \tan ^{-1}(\frac{x}{\pi}) \\ \sin ^{-1}(\frac{x}{\pi}) & \cot ^{-1}(\pi x) \end{bmatrix} -\frac{1}{\pi} \begin{bmatrix} -\cos ^{-1}(x \pi) & \tan ^{-1}(\frac{x}{\pi}) \\ \sin ^{-1}(\frac{x}{\pi}) & -\tan ^{-1}(\pi x) \end{bmatrix} \\ & =\frac{1}{\pi} \begin{bmatrix} \sin ^{-1}(x \pi)+\cos ^{-1}(x \pi) & \tan ^{-1}(\frac{x}{\pi})-\tan ^{-1}(\frac{x}{\pi}) \\ \sin ^{-1}(\frac{x}{\pi})-\sin ^{-1}(\frac{x}{\pi}) & \cot ^{-1}(\pi x)+\tan ^{-1}(\pi x) \end{bmatrix} \\ & =\frac{1}{\pi} \begin{bmatrix} \frac{\pi}{2} & 0 \\ 0 & \frac{\pi}{2} \end{bmatrix} \quad \begin{bmatrix} \because \sin ^{-1} x+\cos ^{-1} x=\frac{\pi}{2} \\ \tan ^{-1} x+\cot ^{-1} x=\frac{\pi}{2} \end{bmatrix} \\ & =\frac{1}{\pi} \times \frac{\pi}{2} \begin{bmatrix} 1 & 0 \\ 0 & 1 \end{bmatrix} =\frac{1}{2} I \end{aligned} $

Hence, the correct option is $(d)$.

  • Option (a) I:

    • This option is incorrect because the resulting matrix ( A - B ) is not the identity matrix ( I ). The identity matrix ( I ) has 1s on the diagonal and 0s elsewhere, but ( A - B ) results in a matrix with (\frac{1}{2}) on the diagonal and 0s elsewhere.
  • Option (b) O:

    • This option is incorrect because the resulting matrix ( A - B ) is not the zero matrix ( O ). The zero matrix ( O ) has all elements equal to 0, but ( A - B ) results in a matrix with (\frac{1}{2}) on the diagonal and 0s elsewhere.
  • Option (c) 2I:

    • This option is incorrect because the resulting matrix ( A - B ) is not twice the identity matrix ( 2I ). The matrix ( 2I ) would have 2s on the diagonal and 0s elsewhere, but ( A - B ) results in a matrix with (\frac{1}{2}) on the diagonal and 0s elsewhere.

57. If $A$ and $B$ are two matrices of the order $3 \times m$ and $3 \times n$, respectively and $m=n$, then the order of matrix $(5 A-2 B)$ is

(a) $m \times 3$

(b) $3 \times 3$

(c) $m \times n$

(d) $3 \times n$

Show Answer

Solution

As we know that the addition and subtraction of two matrices is only possible when they have same order. It is also given that $m=n$.

$\therefore$ Order of $(5 A-2 B)$ is $3 \times n$

Hence, the correct option is $(d)$.

  • Option (a) $m \times 3$: This option is incorrect because the order of the resulting matrix $(5A - 2B)$ must match the order of the original matrices $A$ and $B$, which is $3 \times n$. Since $m = n$, the resulting matrix cannot have an order of $m \times 3$.

  • Option (b) $3 \times 3$: This option is incorrect because the order of the resulting matrix $(5A - 2B)$ must match the order of the original matrices $A$ and $B$, which is $3 \times n$. There is no indication that $n$ is equal to 3, so the resulting matrix cannot have an order of $3 \times 3$.

  • Option (c) $m \times n$: This option is incorrect because the order of the resulting matrix $(5A - 2B)$ must match the order of the original matrices $A$ and $B$, which is $3 \times n$. Since $m = n$, the resulting matrix cannot have an order of $m \times n$; it must be $3 \times n$.

58. If $A= \begin{bmatrix} 0 & 1 \\ 1 & 0 \end{bmatrix} $, then $A^{2}$ is equal to

(a) $ \begin{bmatrix} 0 & 1 \\ 1 & 0 \end{bmatrix} $

(b) $ \begin{bmatrix} 1 & 0 \\ 1 & 0 \end{bmatrix} $

(c) $ \begin{bmatrix} 0 & 1 \\ 0 & 1 \end{bmatrix} $

(d) $ \begin{bmatrix} 1 & 0 \\ 0 & 1 \end{bmatrix} $

Show Answer

Solution

Given that $A= \begin{bmatrix} 0 & 1 \\ 1 & 0 \end{bmatrix} $

$ A^{2}=A \cdot A= \begin{bmatrix} 0 & 1 \\ 1 & 0 \end{bmatrix} \begin{bmatrix} 0 & 1 \\ 1 & 0 \end{bmatrix} = \begin{bmatrix} 0+1 & 0+0 \\ 0+0 & 1+0 \end{bmatrix} = \begin{bmatrix} 1 & 0 \\ 0 & 1 \end{bmatrix} $

Hence, the correct option is $(d)$.

  • Option (a): This option suggests that ( A^2 = A ). However, squaring a matrix generally does not yield the original matrix unless it is an idempotent matrix (i.e., ( A^2 = A )). In this case, ( A ) is not idempotent, as shown by the calculation ( A^2 = \begin{bmatrix} 1 & 0 \ 0 & 1 \end{bmatrix} ).

  • Option (b): This option suggests that ( A^2 = \begin{bmatrix} 1 & 0 \ 1 & 0 \end{bmatrix} ). This is incorrect because the calculation of ( A^2 ) involves matrix multiplication, and the resulting matrix does not match this option. The correct result is the identity matrix ( \begin{bmatrix} 1 & 0 \ 0 & 1 \end{bmatrix} ).

  • Option (c): This option suggests that ( A^2 = \begin{bmatrix} 0 & 1 \ 0 & 1 \end{bmatrix} ). This is incorrect because the calculation of ( A^2 ) involves matrix multiplication, and the resulting matrix does not match this option. The correct result is the identity matrix ( \begin{bmatrix} 1 & 0 \ 0 & 1 \end{bmatrix} ).

59. If matrix $A=[a _{i j}] _{2 \times 2}$, where $a _{i j}=1$ if $i \neq j$

$ =0 \text{ if } i=j $

then $A^{2}$ is equal to

(a) I

(b) $A$

(c) 0

(d) None of these

Show Answer

Solution

Given that: $\quad A=[a _{i j}] _{2 \times 2}$

Let

$ \begin{aligned} & A= \begin{bmatrix} a _{11} & a _{12} \\ a _{21} & a _{22} \end{bmatrix} _{2 \times 2} \\ & a _{11}=0 \quad[\because i=j] \\ & a _{12}=1 \quad[\because i \neq j] \\ & a _{21}=1 \quad[\because i \neq j] \\ & a _{22}=0 \quad[\because i=j] \end{aligned} $

$\therefore \quad A= \begin{bmatrix} 0 & 1 \\ 1 & 0 \end{bmatrix} $

Now, $A^{2}=A \cdot A= \begin{bmatrix} 0 & 1 \\ 1 & 0\end{bmatrix} \begin{bmatrix} 0 & 1 \\ 1 & 0\end{bmatrix} = \begin{bmatrix} 0+1 & 0+0 \\ 0+0 & 1+0\end{bmatrix} = \begin{bmatrix} 1 & 0 \\ 0 & 1 \end{bmatrix} =I$

Hence, the correct option is (a).

  • Option (b) is incorrect because ( A^2 \neq A ). Specifically, ( A^2 = I ), which is the identity matrix, not the original matrix ( A ).

  • Option (c) is incorrect because ( A^2 \neq 0 ). The result of ( A^2 ) is the identity matrix ( I ), not the zero matrix.

  • Option (d) is incorrect because there is a correct option among the given choices, which is (a). Therefore, “None of these” is not applicable.

60. The matrix $ \begin{bmatrix} 1 & 0 & 0 \\ 0 & 2 & 0 \\ 0 & 0 & 4 \end{bmatrix} $ is a

(a) Identity matrix

(b) symmetric matrix

(c) skew symmetric matrix

(d) none of these

Show Answer

Solution

Let

$ \begin{aligned} A & = \begin{bmatrix} 1 & 0 & 0 \\ 0 & 2 & 0 \\ 0 & 0 & 4 \end{bmatrix} \\ A^{\prime} & = \begin{bmatrix} 1 & 0 & 0 \\ 0 & 2 & 0 \\ 0 & 0 & 4 \end{bmatrix} =A \end{aligned} $

$A^{\prime}=A$, so $A$ is a symmetric matrix.

Hence, the correct option is (b).

  • Option (a) Identity matrix: An identity matrix is a square matrix in which all the elements of the principal diagonal are ones and all other elements are zeros. The given matrix has diagonal elements 1, 2, and 4, not all ones, so it is not an identity matrix.

  • Option (c) Skew symmetric matrix: A skew symmetric matrix is a square matrix that is equal to the negative of its transpose, i.e., ( A’ = -A ). For the given matrix, ( A’ = A ), which is not equal to (-A). Therefore, it is not a skew symmetric matrix.

  • Option (d) None of these: This option is incorrect because the matrix is indeed a symmetric matrix, as shown in the solution.

61. The matrix $ \begin{bmatrix} 0 & -5 & 8 \\ 5 & 0 & 12 \\ -8 & -12 & 0 \end{bmatrix} $ is a

(a) diagonal matrix

(b) symmetric matrix

(c) skew symmetric matrix

(d) scalar matrix

Show Answer

Solution

Let

$ \begin{aligned} & A= \begin{bmatrix} 0 & -5 & 8 \\ 5 & 0 & 12 \\ -8 & -12 & 0 \end{bmatrix} \\ & A^{\prime}= \begin{bmatrix} 0 & 5 & -8 \\ -5 & 0 & -12 \\ 8 & 12 & 0 \end{bmatrix} \end{aligned} $

$ \Rightarrow \quad A^{\prime}=- \begin{bmatrix} 0 & -5 & 8 \\ 5 & 0 & 12 \\ -8 & -12 & 0 \end{bmatrix} =-A $

$A^{\prime}=-A$, so $A$ is a skew symmetric matrix.

Hence, the correct option is (c).

  • Option (a) diagonal matrix: A diagonal matrix is a square matrix in which all the elements outside the main diagonal are zero. In the given matrix, there are non-zero elements outside the main diagonal, so it is not a diagonal matrix.

  • Option (b) symmetric matrix: A symmetric matrix is a square matrix that is equal to its transpose, i.e., ( A = A^{\prime} ). In the given matrix, the transpose ( A^{\prime} ) is not equal to ( A ), so it is not a symmetric matrix.

  • Option (d) scalar matrix: A scalar matrix is a diagonal matrix in which all the elements on the main diagonal are equal. Since the given matrix is not even a diagonal matrix, it cannot be a scalar matrix either.

62. If $A$ is a matrix of order $m \times n$ and $B$ is a matrix such that $A B^{\prime}$ and $B^{\prime} A$ are both defined, then order of $B$ is

(a) $m \times m$

(b) $n \times n$

(c) $n \times m$

(d) $m \times n$

Show Answer

Solution

Order of matrix $A=m \times n$

Let order of matrix $B$ be $K \times P$

Order of matrix $B^{\prime}=P \times K$

If $AB^{\prime}$ is defined then the order of $AB^{\prime}$ is $m \times K$ if $n=P$

If $B^{\prime} A$ is defined then order of $B^{\prime} A$ is $P \times n$ when $K=m$

Now, order of $B^{\prime}=P \times K$

$\therefore \quad$ Order of $B=K \times P$

$ =m \times n \quad[\because K=m, P=n] $

Hence, the correct option is $(d)$.

  • Option (a) $m \times m$: This option is incorrect because if $B$ were of order $m \times m$, then $B’$ would be of order $m \times m$. For $AB’$ to be defined, the number of columns of $A$ (which is $n$) must equal the number of rows of $B’$ (which is $m$), but this is not necessarily true unless $m = n$. Additionally, for $B’A$ to be defined, the number of columns of $B’$ (which is $m$) must equal the number of rows of $A$ (which is $m$), which is true, but it does not satisfy the condition for $AB’$.

  • Option (b) $n \times n$: This option is incorrect because if $B$ were of order $n \times n$, then $B’$ would be of order $n \times n$. For $AB’$ to be defined, the number of columns of $A$ (which is $n$) must equal the number of rows of $B’$ (which is $n$), which is true. However, for $B’A$ to be defined, the number of columns of $B’$ (which is $n$) must equal the number of rows of $A$ (which is $m$), but this is not necessarily true unless $m = n$.

  • Option (c) $n \times m$: This option is incorrect because if $B$ were of order $n \times m$, then $B’$ would be of order $m \times n$. For $AB’$ to be defined, the number of columns of $A$ (which is $n$) must equal the number of rows of $B’$ (which is $m$), but this is not necessarily true unless $m = n$. Additionally, for $B’A$ to be defined, the number of columns of $B’$ (which is $n$) must equal the number of rows of $A$ (which is $m$), which is true, but it does not satisfy the condition for $AB’$.

63. If $A$ and $B$ are matrices of same order, then $(A B^{\prime}-B A^{\prime})$ is a

(a) skew symmetric matrix

(b) null matrix

(c) symmetric matrix

(d) unit matrix

Show Answer

Solution

Let

$ \begin{aligned} P & =(A B^{\prime}-BA^{\prime}) \\ P^{\prime} & =(A B^{\prime}-BA^{\prime})^{\prime} \\ & =(A B^{\prime})^{\prime}-(BA^{\prime})^{\prime} \\ & =(B^{\prime}) A^{\prime}-(A^{\prime})^{\prime} B^{\prime} \quad[\because(AB)^{\prime}=B^{\prime} A^{\prime}] \\ & =BA^{\prime}-AB^{\prime} \\ & =-(A B^{\prime}-BA^{\prime})=-P \\ P^{\prime} & =-P, \text{ so it is a skew symmetric matrix. } \end{aligned} $

Hence, the correct option is $(a)$.

  • Option (b) null matrix: A null matrix is a matrix in which all elements are zero. The expression (A B^{\prime} - B A^{\prime}) does not necessarily result in a matrix with all zero elements unless (A) and (B) are specifically chosen such that this condition holds. In general, this expression will not yield a null matrix.

  • Option (c) symmetric matrix: A symmetric matrix is a matrix that is equal to its transpose, i.e., (P = P^{\prime}). However, for the given expression (P = A B^{\prime} - B A^{\prime}), we have shown that (P^{\prime} = -P), which means (P) is skew symmetric, not symmetric. Therefore, it cannot be a symmetric matrix.

  • Option (d) unit matrix: A unit matrix (or identity matrix) is a square matrix with ones on the diagonal and zeros elsewhere. The expression (A B^{\prime} - B A^{\prime}) does not necessarily result in such a matrix. The result of this expression depends on the specific entries of matrices (A) and (B) and does not generally produce a unit matrix.

64. If $A$ is a square matrix such that $A^{2}=I$, then

$(A-I)^{3}+(A+I)^{3}-7 A$ is equal to

(a) $A$

(b) $I-A$

(c) $I+A$

(d) $3 A$

Show Answer

Solution

$(A-I)^{3}+(A+I)^{3}-7 A=A^{3}-I^{3}-3 A^{2} I+3 A I^{2}+A^{3}+I^{3}+3 A^{2} I$

$ \begin{aligned} & +3 AI^{2}-7 A \\ = & 2 A^{3}+6 AI-7 A \\ = & 2 A \cdot A^{2}+6 AI-7 A \\ = & 2 AI+6 AI-7 A \\ = & 8 AI-7 A=8 A-7 A \\ = & A \end{aligned} $

$ [A^{2}=I] $

Hence, the correct option is (a).

  • Option (b) $I-A$: This option is incorrect because the expression $(A-I)^{3}+(A+I)^{3}-7 A$ simplifies to $A$, not $I-A$. The detailed simplification shows that the terms involving $I$ cancel out, and the remaining terms simplify to $A$.

  • Option (c) $I+A$: This option is incorrect because the expression $(A-I)^{3}+(A+I)^{3}-7 A$ simplifies to $A$, not $I+A$. The detailed simplification shows that the terms involving $I$ cancel out, and the remaining terms simplify to $A$.

  • Option (d) $3 A$: This option is incorrect because the expression $(A-I)^{3}+(A+I)^{3}-7 A$ simplifies to $A$, not $3A$. The detailed simplification shows that the terms involving $I$ cancel out, and the remaining terms simplify to $A$.

65. For any two matrices $A$ and $B$, we have

(a) $AB=BA$

(b) $AB \neq BA$

(c) $AB=O$

(d) None of the above

Show Answer

Solution

We know that for any two matrices $A$ and $B$, we may have $AB=BA, AB \neq BA$ and $AB=0$, but it is not always true.

Hence, the correct option is $(d)$.

  • Option (a) $AB=BA$: This is incorrect because it is not always true that the product of two matrices is commutative. In general, matrix multiplication is not commutative, meaning $AB$ does not necessarily equal $BA$ for all matrices $A$ and $B$.

  • Option (b) $AB \neq BA$: This is incorrect because there are cases where $AB$ can equal $BA$. While it is true that matrix multiplication is generally not commutative, there exist specific pairs of matrices for which $AB = BA$.

  • Option (c) $AB=O$: This is incorrect because it is not always true that the product of two matrices is the zero matrix. While it is possible for the product of two non-zero matrices to be the zero matrix, it is not a general rule that applies to all matrices.

66. On using elementary column operation $C_2 \to C_2-2 C_1$, in the following matrix equation

$ \begin{bmatrix} 1 & -3 \\ 2 & 4\end{bmatrix} = \begin{bmatrix} 1 & -1 \\ 0 & 1\end{bmatrix} \begin{bmatrix} 3 & 1 \\ 2 & 4 \end{bmatrix} $, we have:

(a) $ \begin{bmatrix} 1 & -5 \\ 0 & 4\end{bmatrix} = \begin{bmatrix} 1 & -1 \\ -2 & 2\end{bmatrix} \begin{bmatrix} 3 & -5 \\ 2 & 0 \end{bmatrix} $

(b) $ \begin{bmatrix} 1 & -5 \\ 0 & 4\end{bmatrix} = \begin{bmatrix} 1 & -1 \\ 0 & 1\end{bmatrix} \begin{bmatrix} 3 & -5 \\ -0 & 2 \end{bmatrix} $

(c) $ \begin{bmatrix} 1 & -5 \\ 2 & 0\end{bmatrix} = \begin{bmatrix} 1 & -3 \\ 0 & 1\end{bmatrix} \begin{bmatrix} 3 & 1 \\ -2 & 4 \end{bmatrix} $

(d) $ \begin{bmatrix} 1 & -5 \\ 2 & 0\end{bmatrix} = \begin{bmatrix} 1 & -1 \\ 0 & 1\end{bmatrix} \begin{bmatrix} 3 & -5 \\ 2 & 0 \end{bmatrix} $

Show Answer

Solution

Given that: $ \begin{bmatrix} 1 & -3 \\ 2 & 4\end{bmatrix} = \begin{bmatrix} 1 & -1 \\ 0 & 1\end{bmatrix} \begin{bmatrix} 3 & 1 \\ 2 & 4 \end{bmatrix} $

Using $C_2 \to C_2-2 C_1$, we get

$ \begin{bmatrix} 1 & -5 \\ 2 & 0 \end{bmatrix} = \begin{bmatrix} 1 & -1 \\ 0 & 1 \end{bmatrix} \begin{bmatrix} 3 & -5 \\ 2 & 0 \end{bmatrix} $

Hence, the correct option is $(d)$.

  • Option (a) is incorrect because the matrix on the right-hand side of the equation does not match the result of applying the column operation ( C_2 \to C_2 - 2C_1 ) to the given matrices. Specifically, the second column of the second matrix on the right-hand side should be (\begin{bmatrix} -5 \ 0 \end{bmatrix}) instead of (\begin{bmatrix} -5 \ 2 \end{bmatrix}).

  • Option (b) is incorrect because the matrix on the right-hand side of the equation does not match the result of applying the column operation ( C_2 \to C_2 - 2C_1 ) to the given matrices. Specifically, the second column of the second matrix on the right-hand side should be (\begin{bmatrix} -5 \ 0 \end{bmatrix}) instead of (\begin{bmatrix} -5 \ 2 \end{bmatrix}).

  • Option (c) is incorrect because the matrix on the left-hand side of the equation does not match the result of applying the column operation ( C_2 \to C_2 - 2C_1 ) to the given matrices. Specifically, the second row of the left-hand side matrix should be (\begin{bmatrix} 2 & 0 \end{bmatrix}) instead of (\begin{bmatrix} 2 & 4 \end{bmatrix}).

67. On using elementary row operation $R_1 \to R_1-3 R_2$ in the following matrix equation

$ \begin{bmatrix} 4 & 2 \\ 3 & 3 \end{bmatrix} = \begin{bmatrix} 1 & 2 \\ 0 & 3 \end{bmatrix} \begin{bmatrix} 2 & 0 \\ 1 & 1 \end{bmatrix} \text{, we have: } $

(a) $ \begin{bmatrix} -5 & -7 \\ 3 & 3\end{bmatrix} = \begin{bmatrix} 1 & -7 \\ 0 & 3\end{bmatrix} \begin{bmatrix} 2 & 0 \\ 1 & 1 \end{bmatrix} $

(b) $ \begin{bmatrix} -5 & -7 \\ 3 & 3\end{bmatrix} = \begin{bmatrix} 1 & 2 \\ 0 & 3\end{bmatrix} \begin{bmatrix} -1 & -3 \\ 1 & 1 \end{bmatrix} $

(c) $ \begin{bmatrix} -5 & -7 \\ 3 & 3\end{bmatrix} = \begin{bmatrix} 1 & 2 \\ 1 & -7\end{bmatrix} \begin{bmatrix} 2 & 0 \\ 1 & 1 \end{bmatrix} $

(d) $ \begin{bmatrix} 4 & 2 \\ -5 & -7\end{bmatrix} = \begin{bmatrix} 1 & 2 \\ -3 & -3\end{bmatrix} \begin{bmatrix} 2 & 0 \\ 1 & 1 \end{bmatrix} $

Show Answer

Solution

We have, $\quad \begin{bmatrix} 4 & 2 \\ 3 & 3\end{bmatrix} = \begin{bmatrix} 1 & 2 \\ 0 & 3\end{bmatrix} \begin{bmatrix} 2 & 0 \\ 1 & 1 \end{bmatrix} $

Using elementary row transformation $R_1 \to R_1-3 R_2$,

$ \begin{bmatrix} -5 & -7 \\ 3 & 3 \end{bmatrix} = \begin{bmatrix} 1 & -7 \\ 0 & 3 \end{bmatrix} \begin{bmatrix} 2 & 0 \\ 1 & 1 \end{bmatrix} $

Hence, the correct option is (a).

  • Option (b) is incorrect because the matrix multiplication $\begin{bmatrix} 1 & 2 \\ 0 & 3\end{bmatrix} \begin{bmatrix} -1 & -3 \\ 1 & 1 \end{bmatrix}$ does not result in $\begin{bmatrix} -5 & -7 \\ 3 & 3\end{bmatrix}$. The correct multiplication would yield a different matrix.

  • Option (c) is incorrect because the matrix $\begin{bmatrix} 1 & 2 \\ 1 & -7\end{bmatrix}$ does not match the required form after applying the elementary row operation $R_1 \to R_1-3 R_2$. Additionally, the matrix multiplication $\begin{bmatrix} 1 & 2 \\ 1 & -7\end{bmatrix} \begin{bmatrix} 2 & 0 \\ 1 & 1 \end{bmatrix}$ does not result in $\begin{bmatrix} -5 & -7 \\ 3 & 3\end{bmatrix}$.

  • Option (d) is incorrect because the matrix $\begin{bmatrix} 4 & 2 \\ -5 & -7\end{bmatrix}$ does not match the result of the elementary row operation $R_1 \to R_1-3 R_2$ applied to the original matrix. Additionally, the matrix multiplication $\begin{bmatrix} 1 & 2 \\ -3 & -3\end{bmatrix} \begin{bmatrix} 2 & 0 \\ 1 & 1 \end{bmatrix}$ does not result in $\begin{bmatrix} 4 & 2 \\ -5 & -7\end{bmatrix}$.

Fillers

68. …… matrix is both symmetric and skew symmetric matrix.

Show Answer

Solution

Null matrix i.e. $ \begin{bmatrix} 0 & 0 \\ 0 & 0\end{bmatrix} $ or $ \begin{bmatrix} 0 & 0 & 0 \\ 0 & 0 & 0 \\ 0 & 0 & 0 \end{bmatrix} $ is both symmetric and skew symmetric matrix.

69. Sum of two skew symmetric matrices is always …… matrix.

Show Answer

Solution

Let $A$ and $B$ be any two matrices

$\therefore$ For skew symmetric matrices

$$ \begin{align*} & A=-A^{\prime} \tag{i}\\ & B=-B^{\prime} \tag{ii} \end{align*} $$

Adding (i) and (ii) we get

$ \Rightarrow \quad \begin{aligned} & A+B=-A^{\prime}-B^{\prime} \\ & A+B=-(A^{\prime}+B^{\prime}), \text{ so } A+B \text{ is skew symmetric } \\ & \text{ matrix. } \end{aligned} $

Hence, the sum of two skew symmetric matrices is always skew symmetric matrix.

70. The negative of a matrix is obtained by multiplying it by ……

Show Answer

Solution

Let A be a matrix

$ \therefore \quad-A=-1 . A $

Hence, negative of a matrix is obtained by multiplying it by -1 .

71. The protuct of any matrix by the scalar …… is the null matrix.

Show Answer

Solution

Let $A$ be any matrix

$ \therefore \quad 0 \cdot A=A \cdot 0=0 $

Hence, the product of any matrix by the scalar $\mathbf{0}$ is the null matrix.

72. A matrix which is not a square matrix is called a …… matrix.

Show Answer

Solution

A matrix which is not a square matrix is called a rectangular matrix.

73. Matrix multiplication is …… over addition.

Show Answer

Solution

Matrix multiplication is distributive over addition. Let A, B and $C$ be any matrices.

So, (i) $A(B+C)=AB+AC$

(ii) $(A+B) C=AC+BC$

74. If $A$ is a symmetric matrix, then $A^{3}$ is a …… matrix.

Show Answer

Solution

Let $A$ be a symmetric matrix

$ \therefore \quad A^{\prime}=A $

$ (A^{3})^{\prime}=(A^{\prime})^{3}=A^{3} \qquad[\because(A^{\prime})^{k}=(A^{k})^{\prime}] $

Hence, if $A$ is a symmetric matrix, then $A^{3}$ is a symmetric matrix.

75. If $A$ is a skew symmetric matrix, then $A^{2}$ is a ……

Show Answer

Solution

If $A$ is a skew symmetric matrix,

$ \therefore \quad \begin{aligned} A^{\prime} & =-A \\ (A^{2})^{\prime} & =(A^{\prime})^{2} \\ & =(-A)^{2} \\ & =A^{2} \end{aligned} $

Hence, $A^{2}$ is a symmetric matrix.

76. If $A$ and $B$ are square matrices of the same order then

(i) $(AB)^{\prime}=$ ……

(ii) $(kA)^{\prime}=$ …… $\qquad (k$ is any scalar quantity)

(iii) $[k(A-B)]^{\prime}=$ ……

Show Answer

Solution

(i) $(AB)^{\prime}=B^{\prime} A^{\prime}$

(ii) $(kA)^{\prime}=k \cdot A^{\prime}$

(iii) $[k(A-B)]^{\prime}=k(A-B)^{\prime}=k(A^{\prime}-B^{\prime})$

77. If $A$ is a skew symmetric, then $k A$ is a …… scalar$(k$ is any scalar)

Show Answer

Solution

If $A$ is a skew symmetric matrix

$ \begin{aligned} \therefore \quad A^{\prime} & =-A \\ (k A)^{\prime} & =k A^{\prime}=k(-A)=-k A \end{aligned} $

Hence, $k A$ is a skew symmetric matrix.

78. If $A$ and $B$ are symmetric matrices, then

(i) $AB-BA$ is a ……

(ii) $BA-2 AB$ is a ……

Show Answer

Solution

(i) Let

$ \begin{matrix} P & =(AB-BA) \\ P^{\prime} & =(AB-BA)^{\prime} \\ & =(AB)^{\prime}-(BA)^{\prime} \\ & =B^{\prime} A^{\prime}-A^{\prime} B^{\prime} & \\ & =BA-AB \quad[\because(AB)^{\prime}=B^{\prime} A^{\prime}] \\ & =-(AB-BA) & {[\because A^{\prime}=A \text{ and } B^{\prime}=B]} \\ & =-P \end{matrix} $

Hence, $(A B-B A)$ is a skew symmetric matrix.

(ii) Let

$ \begin{aligned} Q & =(B A-2 A B) \\ Q^{\prime} & =(B A-2 A B)^{\prime} \\ & =(B A)^{\prime}-(2 A B)^{\prime} \\ & =A^{\prime} B^{\prime}-2(A B)^{\prime} \quad[\because(k A)^{\prime}=k A^{\prime}] \\ & =A^{\prime} B^{\prime}-2 B^{\prime} A^{\prime} \quad \\ & =A B-2 B A \quad[\because A^{\prime}=A \text{ and } B^{\prime}=B] \\ & =-(2 B A-A B) \end{aligned} $

Hence, $(B A-2 A B)$ is neither a symmetric nor a skew symmetric matrix.

79. If $A$ is a symmetric matrix, then $B^{\prime} A B$ is ……

Show Answer

Solution

If $A$ is a symmetric matrix

$ \therefore \quad A^{\prime}=A $

Let

$ \begin{matrix} P & =B^{\prime} AB \\ P^{\prime} & =(B^{\prime} AB)^{\prime} \\ & =B^{\prime} A^{\prime}(B^{\prime})^{\prime} & \\ & =B^{\prime} AB \quad[\because(AB)^{\prime}=B^{\prime} A^{\prime}] \\ & {[\because A^{\prime}=A \text{ and }(B^{\prime})^{\prime}=B]} \end{matrix} $

$ \therefore \quad P^{\prime}=P $

So, $P$ is a symmetric matrix.

Hence, $B^{\prime} AB$ is a symmetric matrix.

80. If $A$ and $B$ are symmetric matrices of same order, then $A B$ is symmetric if and only if ……

Show Answer

Solution

Given that

$A^{\prime}=A$

and

$B^{\prime}=B$

Let

$ \begin{aligned} P & =AB \\ P^{\prime} & =(AB)^{\prime} \\ & =B^{\prime} A^{\prime} \\ P^{\prime} & =BA \\ & =P \end{aligned} $

$ [\because A^{\prime}=A \text{ and } B^{\prime}=B] $

Hence, $AB$ is symmetric if and only if $\mathbf{A B}=\mathbf{B A}$.

81. In applying one or more row operations while finding $A^{-1}$ by elementary row operations, we obtain all zeros in one or more, then $A^{-1}$ ……

Show Answer

Solution

$A^{-1}$ does not exist if we apply one or more row operations while finding $A^{-1}$ by elementary row operations, obtain all zeroes in one or more rows.

True/False

82. A matrix denotes a number.

Show Answer

Solution

False.

A matrix is an array of elements, numbers or functions having rows and columns.

83. Matrices of any order can be added.

Show Answer

Solution

False.

The matrices having same order can only be added.

84. Two matrices are equal if they have same number of rows and same number of columns.

Show Answer

Solution

False.

The two matrices are said to be equal if their corresponding elements are same.

85. Matrices of different orders can not be subtracted.

Show Answer

Solution

True.

For addition and subtraction, the order of the two matrices should be same.

86. Matrix addition is associative as well as commutative.

Show Answer

Solution

True.

If $A, B$ and $C$ are the matrices of addition then

$ \begin{align*} A+(B+C) & =(A+B)+C \tag{associative}\\ A+B & =B+A \end{align*} $

(commutative)

87. Matrix multiplication is commutative.

Show Answer

Solution

False.

Since $A B \neq B A$ if $A B$ and $B A$ are well defined.

88. A square matrix where every element is unity is called an identity matrix.

Show Answer

Solution

False.

Since, in identity matrix all the elements of principal diagonal are unity rest are zero.

e.g., $\quad A= \begin{bmatrix} 1 & 0 & 0 \\ 0 & 1 & 0 \\ 0 & 0 & 1 \end{bmatrix} =I_3$

89. If $A$ and $B$ are two square matrices of the same order, then $A+B=B+A$.

Show Answer

Solution

True.

If $A$ and $B$ are square matrices then their addition is commutative i.e., $A+B=B+A$.

90. If $A$ and $B$ are two matrices of the same order, then $A-B=$ $B-A$.

Show Answer

Solution

False.

Since subtraction of any two matrices of the same order is not commutative i.e., $A-B \neq B-A$.

91. If matrix $AB=O$, then $A=O$ or $B=O$ or both $A$ and $B$ are null matrices.

Show Answer

Solution

False.

Since for any two non-zero matrices A and B, we may get $AB=0$.

92. Transpose of a column matrix is a column matrix.

Show Answer

Solution

False.

Transpose of a column matrix is a row matrix.

e.g., $A= \begin{bmatrix} 2 \\ 3 \\ 5\end{bmatrix} _{3 \times 1} \quad \therefore A^{\prime}= \begin{bmatrix} 2 & 3 & 5 \end{bmatrix} _{1 \times 3}$

93. If $A$ and $B$ are two square matrices of the same order, then $AB=BA$.

Show Answer

Solution

False.

For two square matrices $A$ and $B, AB=BA$ is not always true.

94. If each of the three matrices of the same order are symmetric, then their sum is a symmetric matrix.

Show Answer

Solution

True.

Let $A, B$ and $C$ be three matrices of the same order.

Given that $A^{\prime}=A, B^{\prime}=B$ and $C^{\prime}=C$

Let

$ \begin{aligned} P & =A+B+C \\ P^{\prime} & =(A+B+C)^{\prime} \\ & =A^{\prime}+B^{\prime}+C^{\prime} \\ & =A+B+C \\ & =P \end{aligned} $

$ \Rightarrow \quad P^{\prime}=(A+B+C)^{\prime} $

So, $A+B+C$ is also a symmetric matrix.

95. If $A$ and $B$ are any two matrices of the same order, then $(A B)^{\prime}=A^{\prime} B^{\prime}$.

Show Answer

Solution

False.

Since $(A B)^{\prime}=B^{\prime} A^{\prime}$.

96. If $(A B)^{\prime}=B^{\prime} A^{\prime}$, where $A$ and $B$ are not square matrices, then number of rows in $A$ is equal to number of columns in $B$ and number of columns in $A$ is equal to number of rows in $B$.

Show Answer

Solution

True.

Let $A=[a _{i j}] _{m \times n}$ and $B=[b _{i j}] _{p \times q}$

$AB$ is defined when $n=P$

$\therefore \quad$ Order of $AB=m \times q$

$\Rightarrow \quad$ Order of $(AB)^{\prime}=q \times m$

Order of $B^{\prime}$ is $q \times p$ and order of $A^{\prime}$ is $n \times m$

$\therefore B^{\prime} A^{\prime}$ is defined when $P=n$

and the order of $B^{\prime} A^{\prime}$ is $q \times m$

Hence, order of $(A B)^{\prime}=$ Order of $B^{\prime} A^{\prime}$ i.e., $q \times m$.

97. If $A, B$ and $C$ are square matrices of same order, then $A B=A C$ always implies that $B=C$.

Show Answer

Solution

False.

Let $A= \begin{bmatrix} 1 & 0 \\ 0 & 0\end{bmatrix} , B= \begin{bmatrix} 0 & 0 \\ 2 & 0\end{bmatrix} $ and $C= \begin{bmatrix} 0 & 0 \\ 3 & 4 \end{bmatrix} $

$ \begin{matrix} \therefore & AB & = \begin{bmatrix} 1 & 0 \\ 0 & 0 \end{bmatrix} \begin{bmatrix} 0 & 0 \\ 2 & 0 \end{bmatrix} = \begin{bmatrix} 0 & 0 \\ 0 & 0 \end{bmatrix} \\ AC & = \begin{bmatrix} 1 & 0 \\ 0 & 0 \end{bmatrix} \begin{bmatrix} 0 & 0 \\ 3 & 4 \end{bmatrix} = \begin{bmatrix} 0 & 0 \\ 0 & 0 \end{bmatrix} \end{matrix} $

Here $A B=A C=0$ but $B \neq C$.

98. $AA^{\prime}$ is always a symmetric matrix of any matrix $A$.

Show Answer

Solution

True.

Let

$ \begin{aligned} P & =AA^{\prime} \\ P^{\prime} & =(AA^{\prime})^{\prime} \\ & =(A^{\prime})^{\prime} \cdot A^{\prime} \\ & =AA^{\prime} \\ & =P \end{aligned} $

So, $P$ is symmetric matrix.

Hence, $AA^{\prime}$ is always a symmetric matrix.

99. If $A= \begin{bmatrix} 2 & 3 & -1 \\ 1 & 4 & 2\end{bmatrix} $ and $B= \begin{bmatrix} 2 & 3 \\ 4 & 5 \\ 2 & 1 \end{bmatrix} $ then $AB$ and $BA$ are defined and equal.

Show Answer

Solution

False. $A= \begin{bmatrix} 2 & 3 & -1 \\ 1 & 4 & 2\end{bmatrix} $ and $B= \begin{bmatrix} 2 & 3 \\ 4 & 5 \\ 2 & 1 \end{bmatrix} $

Since $A B$ is defined

$ \begin{aligned} \therefore \quad AB & = \begin{bmatrix} 2 & 3 & -1 \\ 1 & 4 & 2 \end{bmatrix} \begin{bmatrix} 2 & 3 \\ 4 & 5 \\ 2 & 1 \end{bmatrix} \\ & = \begin{bmatrix} 4+12-2 & 6+15-1 \\ 2+16+4 & 3+20+2 \end{bmatrix} = \begin{bmatrix} 14 & 20 \\ 22 & 25 \end{bmatrix} \end{aligned} $

BA is also defined.

$ \begin{aligned} \therefore \quad BA & = \begin{bmatrix} 2 & 3 \\ 4 & 5 \\ 2 & 1 \end{bmatrix} \begin{bmatrix} 2 & 3 & -1 \\ 1 & 4 & 2 \end{bmatrix} \\ & = \begin{bmatrix} 4+3 & 6+12 & -2+6 \\ 8+5 & 12+20 & -4+10 \\ 4+1 & 6+4 & -2+2 \end{bmatrix} = \begin{bmatrix} 7 & 18 & 4 \\ 13 & 32 & 6 \\ 5 & 10 & 0 \end{bmatrix} \end{aligned} $

So $AB \neq BA$

100. If $A$ is a skew symmetric matrix, then $A^{2}$ is a symmetric matrix.

Show Answer

Solution

True.

$ \begin{matrix} (A^{2})^{\prime} & =(A^{\prime})^{2} & \\ & =[-A]^{2} & {[\because A^{\prime}=-A]} \\ & =A^{2} & \end{matrix} $

So, $A^{2}$ is a symmetric matrix.

101. $(A B)^{-1}=A^{-1} B^{-1}$ where $A$ and $B$ are invertible matrices satisfying commutative property with respective to multiplication.

Show Answer

Solution

True.

If $A$ and $B$ are invertible matrices of the same order

$\therefore$ $(A B)^{-1}$ $=(B A)^{-1}$ $[\because A B=B A]$
But $(A B)^{-1}$ $=A^{-1} B^{-1}$
$\therefore$ $(B A)^{-1}$ $=B^{-1} A^{-1}$
So $A^{-1} B^{-1}$ $=B^{-1} A^{-1}$

$\therefore A$ and $B$ satisfy commutative property w.r.t. multiplication.



Table of Contents